Vous êtes sur la page 1sur 43

Criminal Law Review- Midterm Consolidated Cases

PEOPLE v PETRUS YAU AND SUSANA YAU y SUMOGBA; G.R. No. 208170; August 20, 2014

I. The elements of Kidnapping For Ransom under Article 267 of the RPC, as amended by R.A. No. 7659, are as
follows:

(a) intent on the part of the accused to deprive the victim of his liberty;

(b) actual deprivation of the victim of his liberty; and

(c) motive of the accused, which is extorting ransom for the release of the victim.

All of the foregoing elements were duly established by the testimonial and documentary evidences for the
prosecution in the case at bench. First, Petrus is a private individual. Second, Petrus kidnapped Alastair by using sleeping
substance which rendered the latter unconscious while inside a taxicab driven by the said accused-appellant. Third,
Petrus took and detained Alastair inside the house owned by him and Susana Yau in Bacoor, Cavite, where said victim
was handcuffed and chained, and hence, deprived of his liberty. Fourth, Alastair was taken against his will. And fifth,
Petrus made demands for the delivery of a ransom in the amount of US$600,000.00 for the release of the victim.

II. Criminal liability of each accused-appellant; As a Principal; As an Accomplice

there is no doubt that Petrus is liable as principal of the crime of kidnapping for ransom. Susana, on the other
hand, is liable only as an accomplice to the crime as correctly found by the lower courts. It must be emphasized that there
was no evidence indubitably proving that Susana participated in the decision to commit the criminal act.

Jurisprudence is instructive of the elements required, in accordance with Article 18 of the RPC, in order that a person may
be considered an accomplice, namely;

(1) that there be a community of design; that is, knowing the criminal design of the principal by direct
participation, he concurs with the latter in his purpose;

(2) that he cooperates in the execution by previous or simultaneous act, with the intention of supplying material
or moral aid in the execution of the crime in an efficacious way; and

(3) that there be a relation between the acts done by the principal and those attributed to the person charged
as accomplice.

In the case at bench, Susana knew of the criminal design of her husband, Petrus, but she kept quiet and never reported
the incident to the police authorities. Instead, she stayed with Petrus inside the house and gave food to the victim or
accompanied her husband when he brought food to the victim. Susana not only countenanced Petrus’ illegal act, but also
supplied him with material and moral aid. It has been held that being present and giving moral support when a crime is
being committed make a person responsible as an accomplice in the crime committed. As keenly observed by the RTC,
the act of giving food by Susana to the victim was not essential and indispensable for the perpetration of the crime of
kidnapping for ransom but merely an expression of sympathy or feeling of support to her husband. Moreover, this Court is
guided by the ruling in People v. De Vera, where it was stressed that in case of doubt, the participation of the offender will
be considered as that of an accomplice rather than that of a principal.

III. Penalty

With respect to the penalty, the Court finds that the RTC was correct in imposing the penalty of reclusion perpetua
without eligibility of parole against Petrus as principal in the charge of kidnapping for ransom in view of R.A. No.
9346, prohibiting the death penalty. Also, the Court finds that the penalty of eight (8) years and one (1) day of prision
mayor, as minimum, to twelve (12) years and ten (10) months of reclusion temporal, as maximum, meted out
against Susana, an accomplice, to be proper.

The entire amount of the civil liabilities should be apportioned among all those who cooperated in the commission of the
crime according to the degrees of their liability, respective responsibilities and actual participation. Accordingly, Petrus
should shoulder a greater share in the total amount of damages than Susana who was adjudged only as an accomplice.
(People v. Montesclaros)

FACTS:

1. Petrus and Susana were charged with the crime of Kidnapping For Ransom of private complainant Alastair
Onglingswam, who was put into sleep while riding a taxi (White Toyota taxi cab with plate number PVD-115)
bound to San Juan, Manila. Subsequently, Chau (Alastair girlfriend) received an email from the purported
kidnapper demanding US$2,000.00;

2. During twenty-two (22) days of captivity, he was allowed to communicate with his family almost daily to prove that
he was still alive and was served with meals almost five times a day either by Petrus or the other accused Susan
Yau;
3. The police were able to chance upon the said vehicle. they followed it, then flagged it down and approached the
driver. The driver was asked to scroll down his window and was told that the vehicle was being used to victimize
foreign nationals. Appellant did not offer to make any comment. Hence, this prompted the officers to ask for his
name and since he answered that he was Petrus Yau, a British national, they asked him for his driver’s license
and car registration but appellant was not able to produce any. Since he could not produce any driver’s license
and car registration, they were supposed to bring him to the police station for investigation, however, when shown
a picture of private complainant and asked if he knew him, he answered that the man is being kept in his house.
He was immediately informed that he was being placed under arrest for kidnapping private complainant Alastair
Onglingswam after being informed of his constitutional rights. rescue operations of the victim proceeded
thereafter;

4. When the police proceeded inside the house they found a man sitting on the floor chained and handcuffed. The
man later identified himself as Alastair Onglingswam;

5. RTC finds them guilty beyond reasonable doubt and rejected the twin defenses of alibi and frame-up submitted by
Petrus and Susana because the same were unsubstantiated by clear and convincing evidence

6. The CA affirmed the conviction of Petrus and Susana, Hence this assignment of errors.

ISSUES

 The focus of this case is the degree of responsibility of each accused-appellant for the crime of kidnapping for
ransom.

Case no. 2
People of the Philippines v. Gonzales
183 SSCRA 309, March 19, 1990

Doctrine: The commission of a felony under Art. 3 of the Revised Penal Code requires that an act a punishable act or
omission must be committed, and that it must be committed with deceit and/or fault.

FACTS:
The appellant is appealing to the court regarding his participation in the killing of a certain Loly Penacerrada. He
claims that he did not participate in the killing based on the claim that he was not present in the said act.
The antecedent facts are as follows:
At around 9:00 p.m. of February 21, 1981, Bartolome Paja, barangay captain of Brgy. Tipacla, Ajuy, Iloilo, was
awakened by two of the accused. Paja learns that Fausta killed their landlord, Lloyd Penacerrada, and would like
to surrender to authorities. Knife used in killing was seen, and blood was found smeared on Fausta’s dress.
-Paja immediately ordered a nephew to take spouses to the police at the Municipal Hall in Poblacon, Ajay, where
the couple informed the police on duty of the incident.
-Several patrolmen, along with Paja and Augusto proceeded to the residence at Sitio Nabitasan where the killing
incident allegedly occurred, and found the body of the deceased, clad in underwear, sprawled face down inside
the bedroom.
-Group stayed for an hour in which the scene was inspected, and a rough sketch of the area was made.
-The next day, a patrolman, accompanied by a photographer, went back to the scene for further investigations.
Fausta was brought back to the police station.
-The autopsy of the deceased was performed at 11:20 a.m. Report shows the following:
-Sixteen wounds: five fatal as they penetrated the internal organs
-Multiple puncture, stab, incision, and lacerated wounds
-The day after the autopsy, Augusto appeared before the sub-station and voluntarily surrendered to Police
Corporal Sazon for detention and protective custody for having been involved in the killing of the deceased.
Augusto requests to be taken to where Fausta was already detained.
Based on the investigations conducted, an information for murder dated August 26, 1981, was filed by the
Provincial Fiscal of Iloilo against the spouses. However, they pleaded ‘not guilty.’ Before the trial, however, a certain Jose
Huntoria presented himself to the wife of the deceased. Huntoria claims to be a witness of the killing, and on October 6,
1981, volunteers as a witness for the prosecution. A reinvestigation of the case was called, in which several more were
filed as accused, including the appellant. All the accused except for Lenida pleaded not guilty
At the trial, the prosecution presented Dr. Jesus Rojas, the physician who conducted the autopsy on the body,
Paja, the patrolmen and constabulary members who joined in the investigation, the widow, and Huntoria.
Dr. Rojas testified that he performed the autopsy at around 11:20 a.m. on Feb. 1981 after the deceased was taken
to the municipal hall. He found 4 puncture wounds, 7 stab wounds, 4 incisions, and1 laceration; five of these were fatal
wounds. Rojas admitted one of two possibilities:-
-Only one weapon might have caused all the wounds-
-Multiple instruments were used due to the number and different characteristics

The substance of the prosecution’s case rested on Huntoria’s alleged eyewitness account of theincident, which
was as follows:
-Testified on July 27, 1982; at 5 pm on Feb. 21, 1981, he left his work at Brgy. Central, and walked home,
taking a short-cut.
-While passing at the vicinity of the Gonzales spouses’ home at around 8:00 pm, he heard cries for help.
Curiosity prompted him to approach the place where the shouts were from.
-15-20 m away from the scene, he hid himself behind a clump of banana trees, and saw all the accused
ganging upon the deceased near a threshing platform. He said he clearly recognized all the accused as
the place was awash in moonlight.
-After stabbing and hacking the victim, the accused lifted his body and carried it to the house. Huntoria
then left home. Upon reaching his house, he related what he saw to his wife and mother before going to
sleep.
-Eight months after the incident, bothered by his conscience and the fact that his father was a tenant of
the deceased, he thought of helping the widow. Out of his own volition, he travelled to the widow’s house,
and related to her what he saw
Except Fausta who admitted killing the deceased as he was trying to rape her, the rest denied participation in the
crime. The appellant claimed that he was asleep in his house which was one kilometer away from the scene of the crime,
and he knew of the crime only when his grandchildren went to his house that night
The trial court disregarded the version of the defense; it believed the prosecution’s version.
On appeal to the Court of Appeals, the appellant contended that the trial court erred in convicting him on the basis
of the testimony of the lone witness, and in not appreciating his defense of alibi. The Court found no merit in the errors,
and rejected defense of alibi. Worsening this is that the appellate court found the sentence erroneous, and upgraded the
penalty to that of murder —reclusion temporal/death. The case is now brought upon certification by the Court of Appeals,
hence the appeal
ISSUE:
Whether or not the client, under the evidence presented, has committed the felony of murder.

HELD:
NO. Court’s analysis of the evidence:
-Investigation conducted left much to be desired. Centeno gave the date of commission as March 21, 1981. The
sketch made was troubling, as it did not effectively indicate the extent of the blood stains in the scenes of crime. This
would have added a lot of weight to any one of the versions of the incident
-Sazon, who claimed that Gonzales surrendered to him, failed to state clearly the reason for the surrender. It may
even be possible that Augusto surrendered just so he could be safe from the victim’s kin. Sazon also admitted that
Augusto never mentioned to him the participation of other persons in the killing.
-Rojas’ statement showed two possibilities for the killing. Fausta’s admission that she was the only killer is
plausible. Furthermore, there were only five fatal wounds, which will be discussed later.
-Huntoria’s testimony, of which the prosecution’s argument solely rests, needs to be examined further. Huntoria’s
claims in his testimony did not exactly match with those from his cross-examination. He first claimed that he recognized
the people involved. However, in the cross-examination, he “only saw flashes.” This implies that he may not have
recognized anyone at all.

On the criminal liability of the appellant:


-There is nothing in the findings or the evidence that establishes the criminal liability of the appellant as a principal
for direct participation under Art. 17, para. 1 of the Revised PenalCode.
-Furthermore, there is nothing in the findings or evidence that inculpates him by inducement, under paragraph 2 of
the same article. Based on the definition of felonies in Art. 3 of the Revised Penal Code, the prosecution’s evidence could
not establish intent nor fault. Recall that the elements of felonies include:
-An act or omission
-Act or omission must be punishable
-Act is performed or omission incurred by deceit or fault
-The lone witness could not properly establish any acts or omissions done by the appellant. He stated that he
does not know who hacked or stabbed the victim, thus implying that he does not know what the appellant did. With this,
the essential elements of felonies may not even be present.
-Furthermore, the fact that there were five stab wounds and six accused would imply tha one of them may not
have caused a grave wound (especially given the statement of the physician). This may have been the appellant, and
given that there is no evidence that the appellant caused any of the wounds, coupled with the prosecution’s failure to
prove the presence of conspiracy (that is, how many people actually took part in the killing), it weakens the arguments
against the appellant. On the lone witness:-

Huntoria’s credibility as a witness is tarnished by two points:


1. He came out eight months after the killing. He claims that he feared for his life, but there was no proof that he
was being threatened, nor was the length of time reasonable given the circumstances.
2. He is not exactly a disinterested/neutral witness. He admitted to being a tenant of the deceased, and stated
that one of the reasons why he testified was because the victim was his landlord.-
-Under our socioeconomic set-up, a tenant owes the source of his livelihood from his landlord. As such, they
would do everything to get the landlords to their favor. Posing as a witness would have been a convenient way to do this,
especially as he ceased to be employed as early as May 1981.
Finally, based on Philippine customs and traditions, it is unlikely for the appellant to be in the scene of the crime,
as under our family culture, aging parents are usually sheltered and insulated from possible harm. It is improbable for the
accused to bring their aging father when they were clearly in better shape than he was, and it was unlikely for the
appellant to offer his services as they were more or less enough to handle what could have been a perceived enemy.
Although alibi is a weak defense, in cases like this where the participation of the appellant is not clear, it may be
considered. In light of the evidence on record, it may be sufficient for an acquittal.
Decision of the CA is reversed and set aside. Appellant acquitted

(case No. 3 is in pdf format)


CASE No. 4
U.S. vs. DELOS REYES
G.R. No. 504 – September 16, 1902

DOCTRINE:
There can be no conviction under the Penal Code where by reason of a mistake of fact the intention to commit the crime
does not exist. However, the diligence with which the law requires the individual at all times to govern his conduct varies
with the nature of the situation in which he is to perform. In a matter so important to the good order of society as that in
question, where the consequences of a mistake are necessarily so serious, nothing less than the highest degree of
diligence will satisfy the standard prescribed by the law.
FACTS:
Julian Gonzales and Accused Tomasa Delos Reyes were married in 1897. After some time, the two separated, with
Gonzales living elsewhere while Delos Reyes stayed at their home where Gonzales’ mother also lived. On July 12,1900,
Delos Reyes remarried to Ramon Martinez. Thus, Gonzales filed for a case of bigamy against his wife.
As her defense, she testified that she honestly believed that her husband was dead when she married Martinez. That
sometime after they separated, the mother of Gonzales informed her that her son was dead and she believed this to be
true.
As witness, Gonzales testified that after he left their house, he would still visit his relatives who were left living in their
house almost every day until a few days before the trial of this case.
ISSUE:
Was acquittal of the accused for the crime of bigamy proper? – YES
RULING:
Acquittal for bigamy was proper, however, accused should be convicted under Art. 568 (Old Penal Code: reckless
imprudence)
U.S. vs. Ah Chong (15 Phil. 488)
Doctrine/Topic: Art. 3; Mental element (Mens rea) - Deliberate intent (Dolo) - Mistake of fact
Had the facts been as Ah Chong believed them to be, he would have been justified in killing the intrude. Par. 1, Art XI of
the Revised Penal Code (RPC) provides that in order for the act to be justified, the requisites must be present.
Requisites (Honest Mistake of Fact):
1) Unlawful agression on the part of the victim
2) Reasonable necessity of the means employed to prevent or repel the unlawful aggression
3) Lack of sufficient provocation on the part of the person defending himself
If the victim was really a robber, forcing his way into the room of Ah Chong, there would have been unlawful aggression,
there would have been a necessity on the part of Ah Chong to defend himself and/or his home and the knife would have
been a reasonable means to prevent or repel such aggression.
The act done by Ah Chong was merely an act done due to Honest Mistake of Fact. The Court acquits Ah Chong.
FACTS:
The defendant, Ah Chong, was employed as a cook at “Officers’ quarters. On the night, the defendant, who had received
for the night, was suddenly awakened by some trying to force open the door of the room. He sat up in bed and called out
twice, “Who is there?” He heard no answer and was convinced by the noise at the door that it was being pushed open by
someone bent upon forcing his way into the room. The defendant, fearing that the intruder was a robber or a thief, leaped
to his feet and called out: “If you enter the room, I will kill you.” He was struck just above the knee by the edge of the chair
and he thought that the blow had been inflicted by the person who had forced the door open, whom he supposed to be a
burglar. Seizing a common kitchen knife which he kept under his pillow, the defendant struck out wildly at the intruder
who, it afterwards turned out, was his roommate. The roommate eventually died.
ISSUE: Whether or not Ah Chong may be held criminally responsible for murder in the case at bar.
People v Estrada
Doctrines and Decision:
The basic principle in our criminal law is that a person is criminally liable for a felony committed by him. Under the
classical theory on which our penal code is mainly based, the basis of criminal liability is human free will. Man is
essentially a moral creature with an absolutely free will to choose between good and evil. When he commits a felonious or
criminal act (delito doloso), the act is presumed to have been done voluntarily,i.e., with freedom, intelligence and intent.
Man, therefore, should be adjudged or held accountable for wrongful acts so long as free will appears unimpaired.
In the absence of evidence to the contrary, the law presumes that every person is of sound mind and that all acts are
voluntary. The moral and legal presumption under our law is that freedom and intelligence constitute the normal condition
of a person. This presumption, however, may be overthrown by other factors; and one of these is insanity which exempts
the actor from criminal liability.
ART. 12. Circumstances which exempt from criminal liability.The following are exempt from criminal liability:
1. An imbecile or an insane person, unless the latter has acted during a lucid interval.
When the imbecile or an insane person has committed an act which the law defines as a felony (delito), the court shall
order his confinement in one of the hospitals or asylums established for persons thus afflicted, which he shall not be
permitted to leave without first obtaining the permission of the same court.
An insane person is exempt from criminal liability unless he has acted during a lucid interval. If the court therefore finds
the accused insane when the alleged crime was committed, he shall be acquitted but the court shall order his confinement
in a hospital or asylum for treatment until he may be released without danger. An acquittal of the accused does not result
in his outright release, but rather in a verdict which is followed by commitment of the accused to a mental institution.
In the eyes of the law, insanity exists when there is a complete deprivation of intelligence in committing the act. Mere
abnormality of the mental faculties will not exclude imputability. The accused must be so insane as to be incapable of
entertaining a criminal intent. He must be deprived of reason and act without the least discernment because there is a
complete absence of the power to discern or a total deprivation of freedom of the will.
Sec. 12. Suspension of arraignment.The arraignment shall be suspended, if at the time thereof:
(a) The accused appears to be suffering from an unsound mental condition which effectively renders him unable
to fully understand the charge against him and to plead intelligently thereto. In such case, the court shall order his
mental examination and, if necessary, his confinement for such purpose.
(b) x x x.
The arraignment of an accused shall be suspended if at the time thereof he appears to be suffering from an unsound
mental condition of such nature as to render him unable to fully understand the charge against him and to plead
intelligently thereto. Under these circumstances, the court must suspend the proceedings and order the mental
examination of the accused, and if confinement be necessary for examination, order such confinement and examination. If
the accused is not in full possession of his mental faculties at the time he is informed at the arraignment of the nature and
cause of the accusation against him, the process is itself a felo de se, for he can neither comprehend the full import of the
charge nor can he give an intelligent plea thereto.
The fact that accused-appellant was able to answer the questions asked by the trial court is not conclusive evidence that
he was competent enough to stand trial and assist in his defense. Section 12, Rule 116 speaks of an unsound mental
condition that effectively renders [the accused] unable to fully understand the charge against him and to plead intelligently
thereto. It is not clear whether accused-appellant was of such sound mind as to fully understand the charge against him. It
is also not certain whether his plea was made intelligently. The plea of not guilty was not made by accused-appellant but
by the trial court because of his refusal to plead.
Section 12, Rule 116 of the 1985 Rules on Criminal Procedure speaks of a mental examination. The human mind is an
entity, and understanding it is not purely an intellectual process but depends to a large degree upon emotional and
psychological appreciation.Thus, an intelligent determination of an accuseds capacity for rational understanding ought to
rest on a deeper and more comprehensive diagnosis of his mental condition than laymen can make through observation
of his overt behavior. Once a medical or psychiatric diagnosis is made, then can the legal question of incompetency be
determined by the trial court. By this time, the accuseds abilities may be measured against the specific demands a trial
will make upon him.
Facts:
That on or about the 27th day of December 1994 in the City of Dagupan, Philippines, the accused, Roberto Estrada, being
then armed with a butchers knife, with intent to kill one Rogelio Mararac with treachery and committed in a holy place of
worship, did then and there, wilfully, unlawfully and criminally, attack, assault and use personal violence upon the latter by
stabbing him, hitting him on vital parts of his body with the said weapon, thereby causing his death shortly thereafter due
to Cardiorespiratory Arrest, Massive Intrathoracic Hemorrhage, Stab Wound as per Autopsy Report and Certificate of
Death. At the arraignment, accused-appellants counsel, filed an Urgent Motion to Suspend Arraignment and to Commit
Accused to Psychiatric Ward at Baguio General Hospital. It was alleged that accused-appellant could not properly and
intelligently enter a plea because he was suffering from a mental defect; that before the commission of the crime, he was
confined at the psychiatric ward of the Baguio General Hospital in Baguio City. He prayed for the suspension of his
arraignment and the issuance of an order confining him at the said hospital. The motion was opposed by the City
Prosecutor. The trial court, motu proprio, propounded several questions on accused-appellant. Finding that the questions
were understood and answered by him intelligently, the court denied the motion that same day.
Issue:
Whether or not insanity of a person can exempt him from criminal liability and postpone the trial for a case against him.
Whether the judge through intelligent questions can validly determine the mental capacity of a person in determining
whether to continue with the trial and with his liability in a criminal case.

JESUSA CRUZ VS CORRECTIONAL INSTITUTION FOR WOMEN IN MANDALUYONG


G.R. NO 125672 Septmeber 27, 1996

RA 7659, which took effect on December 13, 1993, partly modified the penalties prescribed by R.A. 6425
(Dangerous Drugs Act of 1972), that is, inter alia, where the quantity of prohibited drugs involved is less than 750 grams,
the penalty is reduced to a range of prision correccional to reclusion perpetua and that where the marijuana is less than
250 grams, the penalty to be imposed shall be prision correccional. Moreover, applying the Indeterminate Sentence Law,
the penalty imposable is further reduced to any period within arresto mayor, as minimum term, to the medium period
of prision correccional as the maximum term, there being no aggravating or mitigating circumstances

The petitioner should now be deemed to have served the maximum period imposable for the crime for which she
was convicted, i.e., selling 5.5 grams of dried marijuana leaves. Although her penalty of life imprisonment had already
become final, the beneficial effects of the amendment provided under R.A. 7659 should be extended to petitioner.

FACTS:
Petitoner Jesusa Cruz, presently confined at the Correctional Institution for Women in Mandaluyong City serving
the penalty of life imprisonment imposed upon her as a consequence of her conviction which became final and executory
last March 1, 1993 for violation of Section 4, Article II of R.A 6425, filed, through her counsel, the present petition for
habeas corpus. Petitioner alleges that, as of the date of filing of her herein petition, she has already served five and a half
years of her life sentence. She argues that the penalty of life imprisonment imposed by the trial court is excessive
considering that the marijuana allegedly taken from her was only 5.5 grams or less than 750 grams.

ISSUE: After having served five and a half years of her life sentence, may petitioner -- who was convicted of selling 5.5
grams of prohibited drugs, namely, dried marijuana leaves -- be now entitled to the beneficent penalty provisions of R.A.
7659 and be now released from imprisonment?

Case #7
People of the Philippines v. Daniel Quijada
G.R. Nos. 115008-09. July 24, 1996
Article 14 – Aggravating Circumstances

Ruling:
The unequivocal intent of the second paragraph of Section 1 of P.D. No. 1866 is to respect and preserve homicide or
murder as a distinct offense penalized under the Revised Penal Code and to increase the penalty for illegal possession of
firearm where such a firearm is used in killing a person. Its clear language yields no intention of the lawmaker to repeal or
modify, pro tanto, Articles 248 and 249 of the Revised Penal Code, in such a way that if an unlicensed firearm is used in
the commission of homicide or murder, either of these crimes, as the case may be, would only serve to aggravate the
offense of illegal possession of firearm and would not anymore be separately punished. Indeed, the words of the subject
provision are palpably clear to exclude any suggestion that either of the crimes of homicide and murder, as crimes mala in
se under the Revised Penal Code, is obliterated as such and reduced as a mere aggravating circumstance in illegal
possession of firearm whenever the unlicensed firearm is used in killing a person. The only purpose of the provision is to
increase the penalty prescribed in the first paragraph of Section 1 -- reclusion temporal in its maximum period to reclusion
perpetua -- to death, seemingly because of the accused's manifest arrogant defiance and contempt of the law in using an
unlicensed weapon to kill another, but never, at the same time, to absolve the accused from any criminal liability for the
death of the victim.
Neither is the second paragraph of Section 1 meant to punish homicide or murder with death if either crime is committed
with the use of an unlicensed firearm, i.e., to consider such use merely as a qualifying circumstance and not as an
offense. That could not have been the intention of the lawmaker because the term "penalty" in the subject provision is
obviously meant to be the penalty for illegal possession of firearm and not the penalty for homicide or murder. We
explicitly stated in Tac-an:
There is no law which renders the use of an unlicensed firearm as an aggravating circumstance in
homicide or murder. Under an information charging homicide or murder, the fact that the death
weapon was an unlicensed firearm cannot be used to increase the penalty for the second offense
of homicide or murder to death. The essential point is that the unlicensed character or condition
of the instrument used in destroying human life or committing some other crime, is not included
in the inventory of aggravating circumstances set out in Article 14 of the Revised Penal Code.

In short, there is nothing in P.D. No. 1866 that manifests, even vaguely, a legislative intent to decriminalize homicide or
murder if either crime is committed with the use of an unlicensed firearm, or to convert the offense of illegal possession of
firearm as a qualifying circumstance if the firearm so illegally possessed is used in the commission of homicide or murder.
To charge the lawmaker with that intent is to impute an absurdity that would defeat the clear intent to preserve the law on
homicide and murder and impose a higher penalty for illegal possession of firearm if such firearm is used in the
commission of homicide or murder.

Facts:
On or about the 30th day of December, 1992, in the municipality of Dauis, province of Bohol, Philippines, the accused,
with intent to kill and without any justifiable motive, with treachery and abuse of superior strength, the accused being then
armed with a .38 cal. revolver, while the victim was unarmed, suddenly attacked the victim without giving the latter the
opportunity to defend himself, and with evident premeditation, the accused having harbored a grudge against the victim a
week prior to the incident of murder, did then and there willfully, unlawfully and feloniously attack, assault and shoot
Diosdado Iroy y Nesnea with the use of the said firearm, hitting the latter on his head and causing serious injuries which
resulted to his death; to the damage and prejudice of the heirs of the deceased. That the accused carried and had in his
possession, custody and control a firearm (hand gun) with ammunition, without first obtaining the necessary permit or
license to possess the said firearm from competent authorities which firearm was carried by the said accused outside of
his residence and was used by him in committing the crime of Murder with Diosdado Iroy y Nesnea as the victim.

Accused-appellant Daniel Quijada now appeals from the decision of the Regional Trial Court (RTC) of Bohol convicting
him of the two offenses separately charged in two informations, viz., murder under Article 248 of the Revised Penal Code
and illegal possession of firearm in its aggravated form under P.D. No. 1866, and imposing upon him the penalty of
reclusion perpetua for the first crime and an indeterminate penalty ranging from seventeen years, four months, and one
day, as minimum, to twenty years and one day, as maximum, for the second crime.

Issue:
Whether or not the RTC is correct in convicting the accused of two separate crimes.

People v. Pinto Jr.

JUSTIFYING CIRCUMSTANCE - In the performance of a Public Duty


AGGRAVATING CIRCUMSTANCE - Evident Premeditation, Unlawful Aggression, Night time, Taking Advantage of
Public Position
MOTIVE - Needed to prove Circumstantial Evidence
ABERATTIO ICTUS - Mistake in the Identity
CONSPIRACY - Acting in concert

=================================
Pt. I
=================================

Search warrant was issued for inspecting the house and premises of Francisco Bello on the ground that the police had
probable cause that Bello illegally possessed a garand rifle, a thompson submachinegun and two automatic pistols
1 of 3 teams were walking on the Mariawa road toward the premises, when one PC Romero heard rumbling of a jeep,
heard gunshots and saw flashes of light from the direction of Buenaflor, his team member running towards the same.

On the jeep which passed by the deployed policemen were Fr. Felix Cappellan, Mrs. Zenaida Stilianopolous Tiongson, her
six children and the driver returning from mass, fearing that there might be "people with bad intentions" or hold-uppers, Fr.
Capellan told the driver to go faster. Shots were fired at the jeep.

After the shooting incident, the police pursued the mission searching the premises.

=================================
Pt. II
=================================
Although Bello and his parents, Mr. and Mrs. Anduiza, were not around, the police searched the area and found a
Japanese Springfield rifle, ammunition of a garand rifle, ammunition of a carbine, live ammunition for a .38 caliber pistol
and 380 bullets for an automatic pistol.
Thereafter, the Chief of Police declared the search terminated and the entire searching party left for headquarters

Team 3 was instrued by a "superior officer" "to remain and maintain peace and order in (the) vicinity including Mariawa".
While he and Buenaflor were patrolling the area, at around midnight, they "chanced upon a house" wherein Bello and his
group were staying. They captured four of Bello's bodyguards and tied them to a pili tree with the torn shirt of one of the
captives he saw a man with a bolo in his hand running towards him. As the man was menacingly near him, Pinto shot the
man later identified as Rosalio Andes when he was at a distance of around three meters

Inocencia's husband was about to offer Bello a cup of coffee when she heard a successive burst of gunfire. Bello, who
was the balcony facing the copra kiln ("agonan") with his back towards the pili tree, gradually fell to the floor with his
hands above his head. Then there was another burst of gunfire. From the kitchen, Inocencia rushed to the door from
where she saw a man holding a long firearm, whom she later identified as Pinto, near the pili tree which was around eight
meters from where Bello was, and another man, also holding a gun, crouching near the stairs

Inocencia was about to rush to Bello. Just then a man, whom Inocencia identified as Buenaflor, came up the house,
pointed a gun at Inocencia and her husband and told them to lay flat on the floor.

When Francisco Andes went up the house, he told Inocencia that Rosalio was dead. Inocencia went near the pili tree
where Rosalio's body was, knelt down and asked the man with a long firearm why he killed Rosalio. The man answered
that Rosalio fought back. However, Inocencia did not notice any weapon near Rosalio's body

=================================
Pt. III
=================================

Buenaflor and Pinto were charged: without any justifiable cause or motive, with intent to kill,

with treachery and evident premeditation, by means of a Cal. 45 Thompson Sub-Machine Gun and a US Carbin Inland,
Cal. 30 owned respectively by said accused,

(1) shoot one Rosalio Andes,23y/o. inflicting upon him gunshot wounds causing upon said Rosalio Andes serious and
mortal wounds which led to his instantaneous death.

(2) one for the murder of Richard Tiongson, 9y/o. sustained a gunshot wound at the back, Richard was operated at the
hospital but he died suffering a hemorrage.

(3) another for the frustrated murder of Maria Theresa Tiongson.

(4) Bello died because of "shock secondary to massive hemorrhage due to multiple gunshot wounds.

=================================
ISSUES
=================================

(1) Is the defense of fulfillment of a duty under Article 11(5) of the RPC tenable?
(2) Is there evident premeditation, treachery, night time, public position present as Aggravating Circumstances?
(3) Is the defense of unlawful aggression tenable?
(4) Is the defense of mistake in the identity of those who were shot in the jeep tenable?

=================================
RULING
=================================

(1) Originally set out to perform a legal duty: the service of a search warrant on Bello. In the process, however, appellants
abused their authority resulting in unauthorized and unlawful moves and consequences. Armed with only a search warrant
and the oral order to apprehend Bello, they went beyond the ambit of their mission and deprived Bello and two other
persons of their lives.

The defense has to prove that these two requisites are present: (a) the offender acted in the performance of a duty and (b)
the injury or offense committed be the necessary consequence of the due performance or lawful exercise of such duty. In
the absence of the second requisite, the justification becomes an incomplete one thereby converting it into a mitigating
circumstance under Articles 13 and 69 of the same Code

The police had obtained a search warrant for illegal possession of firearms against Bello even on Christmas day which
was supposed to be a holiday, no such effort was made in securing warrant of arrest for Bello's alleged frustrated killing of
Botin

(2) On Killing Bello and Andres:


Evident premeditation has not been proven beyond reasonable doubt in this case but we find that the appellants indeed
took advantage of their public position in perpetrating the crime.

(3) As regards the unlawful aggression of Rosalio Andes against Pinto, we find that if we are to believe Pinto, we have to
stamp full credibility on his statement alone. Even Buenaflor admitted that he did not see Rosalio Andes attack Pinto.

(4) On the jeep coming towards them was owned by the Anduizas, the appellants acted obviously in the belief that Bello
was its passenger and posthaste they fired upon it even without any inquiry as to the identity of its passengers

Buenaflor's motive for wanting to do away with Bello has been established. Such motive provided a circumstantial
evidence leading to the inference that indeed he fired his gun

All these pieces of circumstantial evidence point to no other inference than that Pinto and Buenaflor fired their guns in
defiance of their superior officer's order only "to find the whereabouts" of Bello

The fact that the victims were different from the ones the appellants intended to injure cannot save them from conviction.
Aberratio ictus or mistake in the identity of the victim carries the same gravity as when the accused zeroes in on his
intended victim. The main reason behind this conclusion is the fact that the accused had acted with such a disregard for
the life of the victim(s) — without checking carefully the latter's identity as to place himself on the same legal plane as one
who kills another willfully, unlawfully and feloniously. Neither may the fact that the accused made a mistake in killing one
man instead of another be considered a mitigating circumstance

It is not even necessary to pinpoint who between Pinto and Buenaflor actually caused the death of Richard or the
wounding of Maria Theresa in the presence of proof beyond reasonable doubt that they acted in conspiracy with each
other.

The lower court is AFFIRMED with MODIFICATIONS that appellants shall solidarily be liable for each of the three murders
they committed and, for the frustrated murder of Maria Theresa Tiongson

People vs. Oanis


Art. 11, no. 5 of RPC
Art. 69 of RPC
Murder
Rule 109, Sec. 2, par. 2

Facts:
In the afternoon of Dec. 24, 1938, The Constabulary Provincial inspector received a telegram stating " escaped convict
Anselmo Balagtas with bailarina and Irene in Cabanatuan get him dead or alive". The Inspector then called his men,
Included is Alberto Galanta, and instructed to arrest Balagtas, and if overpowered, to follow the instruction contained in
the telegram. The same instructions were given to Chief of Polic Oanis, by the Inspector.

The inspector divided the party into 2 groups to locate Balagtas. Galanta and Oanis were group together and proceeded
to Irene's house. Upon reaching the house, Oanis approached brigida Mallare and asked Irenes location.

Defendants then went to the room of Irene and upon seeing a man sleeping with his back towards the door where they
were were, simultaneously or successively fired at him with their revolvers. It turned out that the person shot and killed
was not the notorious Balagtas, but an innocent Serapio Tecson, who was Irene's paramour.

On trial, the testimonies of the defendants were contrary to each other but such was corroborated by Irene's testimony
that the defendants fired at Tecson while he was lying in bed.

The RTC charged the defendants of Homicide through reckless impudence and senteced each an indeterminate penatly
and a fine to pay the heirs of Tecson.

Defendants contended that they incur no criminal liability because they believed that the man was Balagtas and they
acted in innocent mistake of fact in the honest performance of their official duties.

Issue: WON they should be held responsible for the death caused to Tecson?

Held: Yes, they are responsible for tge death of Tecson.

The SC are of the opinion that the crime committed is murder though specially mitigated. The contention of the defendants
that they are not liable because of their defense of "mistake of fact" is not applicable.

In the case of Ah Chong, it was shown that there was an innocent mistake of fact committed without any fault or
carelessness because the accused, having no time or opportunity to make a further inquiry, and being pressed by
circumstances to act immediately, had no alternative but to take the facts as they then appeared to him.

In the instant case, appellants found no circumstances whatsorever which would press them to immediate action. The
person in the room being asleep, appellants had ample time and opportunity to ascertain his identity without hazard to
themselves, and could even effect a bloodless arrest if any reasonable effort to that end had been made.

The SC also held that the crime committed by appellants is not merely criminal negligence, the killing being intentional
and not accidental. Stating that "a deloberate intent to di an unlawful act is essentially inconsistent with the idea of
reckless imprudence.

There is, however, an incomplete justifying circumstance, falling under Art. 11 no. 5 of the RPC. Only one requisite was
present as the "offenders acted in the performance of a duty or in the lawful exercise of a right". Such being the case, and
applying Art 69 od the RPC, the penalty lower by one or two degress than that prescribed by law shall be imposed.

Intod vs. Court of Appeals

FACTS:
• February 4, 1979: Sulpicio Intod, Jorge Pangasian, Santos Tubio and Avelino Daligdig went to Salvador Mandaya's
house and asked him to go with them to the house of Bernardina Palangpangan. Thereafter, they had a meeting with
Aniceto Dumalagan who told Mandaya that he wanted Palangpangan to be killed because of a land dispute between them
and that Mandaya should accompany them. Otherwise, he would also be killed.
• February 4, 1979 10:00 pm: All of them armed arrived at Palangpangan's house and fired at Palangpangan's bedroom
but there was no one in the room.
• RTC: convicted Intod of attempted murder based on the testimony of the witness

ISSUE: W/N Intod is guilty attempted murder since it is an impossible crime under Art. 4 (2)

HELD: YES. petition is hereby GRANTED, the decision of respondent Court of Appeals holding Petitioner guilty of
Attempted Murder is hereby MODIFIED. sentences him to suffer the penalty of six (6) months of arresto mayor, together
with the accessory penalties provided by the law, and to pay the costs

• Art. 4(2). CRIMINAL RESPONSIBILITY. — Criminal Responsibility shall be incurred:


xxx xxx xxx
2. By any person performing an act which would be an offense against persons or property, were it not for the inherent
impossibility of its accomplishment or on account of the employment of inadequate or ineffectual means.
Petitioner contends that, Palangpangan's absence from her room on the night he and his companions riddled it with
bullets made the crime inherently impossible.
• The Revised Penal Code, inspired by the Positivist School, recognizes in the offender his formidability to punish
criminal tendencies in Art. 4(2)
• Legal impossibility occurs where the intended acts, even if completed, would not amount to a crime
• Legal impossibility would apply to those circumstances where
1. the motive, desire and expectation is to perform an act in violation of the law
2. there is intention to perform the physical act
3. there is a performance of the intended physical act
4. the consequence resulting from the intended act does not amount to a crime
o Ex: The impossibility of killing a person already dead
• Factual impossibility occurs when extraneous circumstances unknown to the actor or beyond his control prevent the
consummation of the intended crime – this case
o Ex: man who puts his hand in the coat pocket of another with the intention to steal the latter's wallet and finds the
pocket empty
• United States: where the offense sought to be committed is factually impossible or accomplishment - attempt to
commit a crime; legally impossible of accomplishment - cannot be held liable for any crime

Case # 12
ESCAMILLA V. PEOPLE
G.R. No. 188551 February 27, 2013
HOMICIDE
DOCTRINE: The intent to kill, as an essential element of homicide at whatever stage, may be before or simultaneous with
the infliction of injuries. The evidence to prove intent to kill may consist of, inter alia, the means used; the nature, location
and number of wounds sustained by the victim; and the conduct of the malefactors before, at the time of, or immediately
after the killing of the victim.l1
Petitioner’s intent to kill was simultaneous with the infliction of injuries. Using a gun, he shot the victim in the chest,
petitioner continued to shoot at him three more times, albeit unsuccessfully. While running, the victim saw his nephew in
front of the house and asked for help. The victim was immediately brought to the hospital. The doctor said that the victim
would have died if the latter were not brought immediately to the hospital. All these facts belie the absence of petitioner’s
intent to kill the victim.
FACTS:
1. Around 2am of 01 August 1999, a brawl ensued at the comer of Estrada and Arellano Streets, Manila. Mendol was
about to ride his tricycle at this intersection while facing Arellano Street.
2. Petitioner, who was standing in front of his store, 30 meters away from Mendol, shot the later four times, hitting him
once in the upper right portion of his chest.
3. The victim was brought to the Ospital ng Makati for treatment and survived because of timely medical attention.
4. Assistant City Prosecutor of Manila Fled an Information charging petitioner with frustrated homicide.
5. During trial, the prosecution presented the testimonies of Mendol, Joseph Velasco (Velasco) and Iluminado
Garcelazo(Garcelazo), who all positively identified the petitioner as the shooter of Mendol.
RTC: found petitioner guilty of frustrated homicide.
CA: affirmed the conviction
Issue: Whether or not the CA is correct in affirming the conviction of frustrated homicide?

13. People vs Campuhan [G.R. No. 129433. March 30, 2000]


Ruling: NO. MODIFIED. guilty of ATTEMPTED RAPE and sentenced to an indeterminate prison term of eight (8) years
four (4) months and ten (10) days of prision mayor medium as minimum, to fourteen (14) years ten (10) months and
twenty (20) days of reclusion temporal medium as maximum. Costs de oficio.

• People v. De la Peña: labia majora must be entered for rape to be consummated


• Primo's kneeling position rendered an unbridled observation impossible
• Crysthel made a categorical statement denying penetration but her vocabulary is yet as underdeveloped
• Corazon narrated that Primo had to hold his penis with his right hand, thus showing that he had yet to attain an
erection to be able to penetrate his victim
• the possibility of Primo's penis having breached Crysthel's vagina is belied by the child's own assertion that she
resisted Primo's advances by putting her legs close together and that she did not feel any intense pain but just felt "not
happy" about what Primo did to her. Thus, she only shouted "Ayo'ko, ayo'ko!" not "Aray ko, aray ko!
• no medical basis to hold that there was sexual contact between the accused and the victim.
FACTS:
• April 25, 1996 4 pm: Ma. Corazon P. Pamintuan, mother of 4-year old Crysthel Pamintuan, went to the ground floor of
their house to prepare Milo chocolate drinks for her 2 children. There she met Primo Campuhan, helper of Conrado Plata
Jr., brother of Corazon, who was then busy filling small plastic bags with water to be frozen into ice in the freezer located
at the second floor.
• Then she heard Crysthel cry, "Ayo'ko, ayo'ko!" so she went upstairs and saw Primo Campuhan inside her children's
room kneeling before Crysthel whose pajamas or "jogging pants" and panty were already removed, while his short pants
were down to his knees and his hands holding his penis with his right hand
• Horrified, she cursed "P - t - ng ina mo, anak ko iyan!" and boxed him several times. He evaded her blows and pulled
up his pants. He pushed Corazon aside who she tried to block his path. Corazon then ran out and shouted for help thus
prompting Vicente, her brother, a cousin and an uncle who were living within their compound, to chase the Campuhan
who was apprehended. They called the barangay officials who detained.
• Physical examination yielded negative results as Crysthel ‘s hymen was intact
• Campuhan: Crysthel was in a playing mood and wanted to ride on his back when she suddenly pulled him down
causing both of them to fall down on the floor.
• RTC: guilty of statutory rape, sentenced him to the extreme penalty of death
• Thus, subject to automatic review

ISSUE: W/N it was a consummated statutory rape

PEOPLE OF THE PHILIPPINES, plaintiff-appellee, vs. VIOLETO VILLACORTE, alias BONGING, et al., defendants.
CRISANTO INOFERIO Y ALINDAO alias SANTE, and MARCIANO YUSAY alias MANCING (appeal withdrawn res. of
7/10/67), defendants-appellants. G.R. No. L-21860 February 28, 1974
RULING: Although inherently weak and easily fabricated, the evidence presented by an accused in support of that
defense must be scrutinized with the same care that evidence supporting other defenses deserves. When an accused
puts up the defense of alibi, the court should not at once have a mental prejudice against him. For, taken in the light of all
the evidence on record, it may be sufficient to acquit him, as in the case of appellant Inoferio. He should be acquitted upon
the ground that although his defense, in the nature of an alibi, is inherently a weak defense, it should be considered
sufficient as in this case, to tilt the scale of justice in favor of the accused because the evidence for the prosecution is itself
weak and unconvincing and, therefore, by and large, insufficient to prove the guilt of the accused beyond reasonable
doubt.
FACTS: After closing his shop for the day, Chinese merchant Benito Ching left his store in the public market of Caloocan.
He brought with him a paper bag where all the proceeds of his sale for the day are placed. While walking with his 2
companions, Libantino and Galvez, 4 men accosted them. Two of the men held Ching and Galvez while Labantino
continued to walk in front of them, initially unaware of the commotion. A .45 caliber pistol as pointed at Ching while he was
held, while another man snatched the paper bag from him. When Ching tried to call Labantino’s name for help, the gun
was shot resulting to a gun wound on Ching. Labantino fled, while Galvez finally freed himself from the men holding him.
The men eventually fled too. After falling from the gun shot, Ching was able to walk to home. His common-law wife
brought him to the hospital. He died a day after.
Galvez and Labantino both identified Villacorte as one of the men who accosted them. Upon interrogation, Villacorte
admitted to the crime and identified his companions as "Roque", "Sante" and "Fred".
Consequently, an information for robbery with homicide was filed against Villacorte, Roque Guerrero (“Roque’), Alfredo
Handig (“Fred”) and an alias “Sante” whose identity was not yet ascertained. The information was eventually amended to
implicate Crisanto Inoferio. The trial court acquitted Handig and found Villacorte and Inoferio guilty. Guerrero was
discharged and used as a state witness.
Only Inoferio pursued this appeal. Inoferio testified that he first met Villacorte and Handig while in Police custody for the
investigation. He claims that he knew Guerrero as they played cara y cruz together.
ISSUE: Whether or not Inoferio’s weak alibi could warrant a conviction.

THE PEOPLE OF THE PHILIPPINES vs. AMADEO PERALTA, ET AL.


Doctrine: A conspiracy exists when two or more persons come to an agreement concerning the commission of a felony
and decide to commit it. Generally, conspiracy is not a crime except when the law specifically provides a penalty therefore
as in treason, rebellion and sedition.
Proof of conspiracy. Conspiracy presupposes the existence of a preconceived plan or agreement; however, to
establish conspiracy, "it is not essential that there be proof as to previous agreement to commit a crime, it being sufficient
that the malefactors committed shall have acted in concert pursuant to the same objective." Hence, conspiracy is proved if
there is convincing evidence to sustain a finding that the malefactors committed an offense in furtherance of a common
objective pursued in concert.
Liability of conspirators. A time-honored rule in the corpus of our jurisprudence is that once conspiracy is
proved, all of the conspirators who acted in furtherance of the common design are liable as co-principals. This rule of
collective criminal liability emanates from the ensnaring nature of conspiracy. The concerted action of the conspirators in
consummating their common purpose is a patent display of their evil partnership, and for the consequences of such
criminal enterprise they must be held solidarity liable.
Imposition of multiple penalties where conspirators commit more than one offense. Since in conspiracy, the
act of one is the act of all, then, perforce, each of the conspirators is liable for all of the crimes committed in furtherance of
the conspiracy. Consequently, if the conspirators commit three separate and distinct crimes of murder in effecting their
common design and purpose, each of them is guilty of three murders and shall suffer the corresponding penalty for each
offense.
Legality and practicality of imposing multiple death penalties upon conspirators. An accused who was
charged with three distinct crimes of murder in a single information was sentenced to two death penalties for two murders,
and another accused to thirteen (13) separate death penalties for the 13 killings he perpetrated. Therefore there appears
to be no legal reason why conspirators may not be sentenced to multiple death penalties corresponding to the nature and
number of crimes they commit in furtherance of a conspiracy
Ruling. The evidence on record proves beyond peradventure that the accused acted in concert from the moment they
bolted their common brigade, up until the time they killed their last victim, Santos Cruz. While it is true that Parumog,
Larita and Luna did not participate in the actual killing of Carriego, nonetheless, as co-conspirators they are equally guilty
and collectively liable for in conspiracy the act of one is the act of all. It is not indispensable that a co-conspirator should
take a direct part in every act and should know the part which the others have to perform. Conspiracy is the common
design to commit a felony; it is not participation in all the details of the execution of the crime. All those who in one way or
another help and cooperate in the consummation of a felony previously planned are co-principals. Hence, all of the six
accused are guilty of the slaughter of Carriego, Barbosa and Santos Cruz — each is guilty of three separate and distinct
crimes of murder.
Facts: On February 16, 1958, while the inmates of the penitentiary were preparing to attend Sunday mass, that a fight
between two rival members of the "Sigue-Sigue" and "OXO" gangs occurred in the plaza where the prisoners were
assembled, causing a big commotion. The fight was, however, quelled, and those involved were led away for
investigation, while the rest of the prisoners were ordered to return to their respective quarters. Moments later, another riot
erupted in Bldg. 4, as the inmates of brigade 4-A, mostly "OXO" members and sympathizers, destroyed the lock of their
door and then rampaged from one brigade to another. The invading “OXO” clubbed and stabbed to death Jose Carriego,
an inmate of 4-B. Afterwards, they forcibly opened the door of 4-C and killed two more inmates, namely, Eugenio Barbosa
and Santos Cruz. The three victims sustained injuries which swiftly resulted in their death — before they could be brought
to the hospital.

Issue: Whether or not the commission of the murders was attended with conspiracy?

PEOPLE OF THE PHILIPPINES, plaintiff-appellee, vs. EDWIN DE VERA yGARCIA, RODERICK GARCIA y
GALAMGAM, KENNETH FLORENDO and ELMER CASTRO, accused, EDWIN DE VERA yGARCIA, appellant.

Doctrines:

Conspiracy - It is axiomatic that the prosecution must establish conspiracy beyond reasonable doubt. Mere presence
does not amount to conspiracy– criminal conspiracy must be founded on facts, not on mere surmises or conjectures.

Requisites of Conspiracy:
To prove conspiracy, the prosecution must establish the following three requisites:
(1) that two or more persons came to an agreement;
(2) that the agreement concerned the commission of a crime; and
(3) that the execution of the felony [was] decided upon.

Accomplice,– Defined; Elements.–


The Court has held that an accomplice is “one who knows the criminal design of the principal and cooperates knowingly
or intentionally therewith by an act which, even if not rendered, the crime would be committed just the same.

To hold a person liable as an accomplice, two elements must be present:


(1) the “community of criminal design; that is, knowing the criminal design of the principal by direct participation, he
concurs with the latter in his purpose–; and
(2) the performance of previous or simultaneous acts that are not indispensable to the commission of the crime.

“Conspirator– and “Accomplice,– Distinguished.–


1. Once conspiracy is proven, the liability is collective and not individual. The act of one of them is deemed the act of
all. In the case of an accomplice, the liability is one degree lower than that of a principal.
2. Conspirators and accomplices have one thing in common: they know and agree with the criminal design.
Conspirators, however, know the criminal intention because they themselves have decided upon such course of
action. Accomplices come to know about it after the principals have reached the decision, and only then do they
agree to cooperate in its execution.
3. Conspirators decide that a crime should be committed; accomplices merely concur in it. Accomplices do not
decide whether the crime should be committed; they merely assent to the plan and cooperate in its
accomplishment.
4. Conspirators are the authors of a crime; accomplices are merely their instruments who perform acts not essential
to the perpetration of the offense.

Accomplices; Penalties;
The penalty of an accomplice is one degree lower than that of a principal.

Murder; Aggravating Circumstances; Treachery; Abuse of Superior Strength; Treachery absorbs abuse of
superior strength
Treachery absorbs abuse of superior strength. The trial court found that the killing was attended by treachery, evident
premeditation and abuse of superior strength. Hence, there is only one generic aggravating circumstance, not two.

Facts:
The accused with evident premeditation, treachery and use of superior strength, attack, assault and employ personal
violence upon the person of one FREDERICK CAPULONG y DIZON, hitting him between his eyes and striking him with
the use of a baseball bat in the mouth, thereby inflicting upon him serious and mortal wounds. The trial court ruled that
there was conspiracy and sentenced all of the accused to suffer reclusion perpetua mainly on the testimony of an
eyewitness. Specifically, it based its conclusions on the following facts: appellant was seen with the other accused inside
the victim’s car; the victim was clearly struck with a blunt object while inside the car, and it was unlikely for Florendo to
have done it all by himself; moreover, it was impossible for De Vera and Garcia to have been unaware of Florendo’s dark
design on Roderick.

Such witness, Cacao, testified that he saw Appellant De Vera in the car, where an altercation later occurred. Thereafter,
he saw Florendo drag out of the vehicle an apparently disabled Capulong and shoot the victim in the head moments later.
But, Cacao’s testimony contains nothing that could inculpate appellant De Vera aside from the fact that he was inside the
car, no other act was imputed to him.

Issues:
1. WON the trial court erred in deciding that there is conspiracy?
2. WON the trial court erred in aside from treachery, there are other two aggravating circumstances (evident
premeditation and abuse of superior strength).

Ruling:
1. YES. The prosecution must establish conspiracy beyond reasonable doubt. Here, the bare testimony of Cacao
fails to do so. Cacao’s testimony contains nothing that could inculpate appellant. Aside from the fact that he was
inside the car, no other act was imputed to him. Mere presence does not amount to conspiracy. Criminal
conspiracy must be founded on facts, not on mere surmises or conjectures
2. YES. Treachery absorbs abuse of superior strength. Hence, there is only one generic aggravating circumstance,
not two.

PEOPLE OF THE PHILIPPINES, Appellee, v. REGIE LABAIGA, Appellant.


G.R. No. 202867, July 15, 2013

ARTICLE – PRINCIPLE OF LAW


 Article 6 Consummated, frustrated, and attempted felonies.

 “In Serrano v. People, we distinguished a frustrated felony from an attempted felony in this manner:
1. In a frustrated felony, the offender has performed all the acts of execution which should produce the
felony as a consequence; whereas in an attempted felony, the offender merely commences the commission of
a felony directly by overt acts and does not perform all the acts of execution.
2. In a frustrated felony, the reason for the non-accomplishment of the crime is some cause independent of
the will of the perpetrator; on the other hand, in an attempted felony, the reason for the non-fulfillment of the
crime is a cause or accident other than the offender’s own spontaneous desistance.”

 “In frustrated murder, there must be evidence showing that the wound would have been fatal were it not for timely
medical intervention. If the evidence fails to convince the court that the wound sustained would have caused the
victim’s death without timely medical attention, the accused should be convicted of attempted murder and not
frustrated murder.”

 “A treacherous attack is one in which the victim was not afforded any opportunity to defend himself or resist the attack.
The existence of treachery is not solely determined by the type of weapon used. If it appears that the weapon was
deliberately chosen to insure the execution of the crime, and to render the victim defenseless, then treachery may be
properly appreciated against the accused.”

FACTS

According to the Prosecution, Regie Labiaga shot Gregorio Conde outside the latter’s house. Conde cried for help which
prompted his daughters Judy and Glenelyn to rush towards him. Regie Labiaga shot Judy and told the other accused
Balatong Barcenas and Cristy Demapanag that “she is already dead.”, and they left the vicinity. Judy and Gregorio were
rushed to the hospital wherein Gregorio was treated and fully recovered Judy was declared dead on arrival due to
“cardiopulmonary arrest secondary to Cardiac Tamponade due to gunshot wound”.
According to the defense, Regie Labiaga admitted that he was present during the shooting incident and claimed that he
acted in self-defense when Gregorio challenged him to a fight while armed with a shotgun. He alleged that when he tried
to wrest the gun from Gregorio, the shotgun fired without him knowing whether somebody was hit. Demapanag alleged
that she was 14 km away from the crime scene which was corroborated by her brother.
The accused were charged with Murder with the Use of Unlicensed Firearm. The information stated that “the accused,
conspiring, confederating and helping one another, armed with unlicensed firearm, with deliberate intent and decided
purpose to kill, by means of treachery and with evident premeditation, did then and there willfully, unlawfully and
feloniously attack, assault and shoot Gregorio Conde with said unlicensed firearm, x x x thereby performing all the acts of
execution which would produce the crime of Murder as a consequence, but nevertheless did not produce it by reason of
causes independent of the will of the accused; that is by the timely and able medical assistance rendered to said Gregorio
Conde which prevented his death.”
The RTC acquitted Demapanag due to insufficiency of evidence while Regie Labiaga was convicted of murder and
frustrated murder. CA-Cebu affirmed the conviction.

ISSUES:
Whether Regie Labiaga is guilty of frustrated or attempted murder.

HELD:
In the instant case, it does not appear that the wound sustained by Gregorio Conde was mortal. This was admitted by Dr.
Edwin Figura, who examined Gregorio after the shooting incident.
Since Gregorio’s gunshot wound was not mortal, the SC held that appellant should be convicted of attempted murder and
not frustrated murder. Under Article 51 of the Revised Penal Code, the corresponding penalty for attempted murder shall
be two degrees lower than that prescribed for consummated murder under Article 248, that is, prision correccional in its
maximum period to prision mayor in its medium period.
Case No. 18
People vs. Narvaez
121 SCRA 389
G.R. Nos. L-33466-67
April 20, 1983

Intro:
Defense of one's person or rights is treated as a justifying circumstance under Art. 11, par. 1 of the Revised Penal Code,
but in order for it to be appreciated, the following requisites must occur:
First. Unlawful aggression;
Second. Reasonable necessity of the means employed to prevent or repel it;
Third. Lack of sufficient provocation on the part of the person defending himself (Art. 11, par. 1, Revised Penal Code, as
amended).

In this case, there was an aggression, not on the person of appellant, but on his property rights when the victims angrily
ordered the continuance of the fencing. There was also no provocation at all since the appellant was asleep before the
commission of the crime. However, the second element is not present as the killing was disproportionate to the attack.

Facts:
On August 22, 1968, Narvaez shot Fleischer and Rubia as the two were constructing a fence that would prevent Narvaez
from getting into his house and rice mill. The defendant was taking a nap when he heard sounds of construction and found
fence being made. He tried to stop the group from destroying his house and asked if they could talk things over but
Fleischer responded with "No, gadamit, proceed, go ahead." Defendant lost his "equilibrium," and shot Fleisher and Rubia
with his shotgun. Defendant claims he killed in defense of his person and property but the CFI ruled that
Narvaez was guilty and sentenced him to reclusion perpetua, to indemnify the heirs, and to pay for moral damages.

Issue:
Whether or not the act of killing of the accused is in defense of his person and of his rights, and therefore he should be
exempt from criminal liability.

Flores vs People G.R. No. 181354, February 27, 2013

Article #/Doctrine: To successfully claim self-defense, the accused must satisfactorily prove the concurrence of the
elements of self-defense. Under Article 11 of the Revised Penal Code, any person who acts in defense of his person
or rights does not incur any criminal liability provided that the following circumstances concur: (1) unlawful
aggression; (2) reasonable necessity of the means employed to prevent or repel it; and (3) lack of sufficient
provocation on the part of the person defending himself.

The most important among all the elements is unlawful aggression. "There can be no self-defense, whether complete or
incomplete, unless the victim had committed unlawful aggression against the person who resorted to self-
defense."30 "Unlawful aggression is defined as an actual physical assault, or at least a threat to inflict real imminent injury,
upon a person. In case of threat, it must be offensive and strong, positively showing the wrongful intent to cause injury. It
presupposes actual, sudden, unexpected or imminent danger––not merely threatening and intimidating action. It is
present only when the one attacked faces real and immediate threat to one’s life." 31"Aggression, if not continuous, does
not constitute aggression warranting self-defense." 32

In this case, Flores failed to discharge his burden. Indeed, the nature and number of the gunshot wounds inflicted upon
Jesus further negate the claim of self-defense by the accused. Records show that Jesus suffered four (4) gunshot wounds
in the different parts of his body, specifically: on the medial portion of the left shoulder, between the clavicle and the first
rib; on the left hypogastric region through the upper right quadrant of the abdomen; on the tip of the left buttocks to the tip
of the sacral bone or hip bone; and on the right flank towards the umbilicus.

According to Dr. Ruben Escueta, who performed the autopsy on the victim, the latter died of massive intra-abdominal
hemorrhage due to laceration of the liver. 36 If there was any truth to Flores’ claim that he merely acted in self-defense, his
first shot on Jesus’ shoulder, which already caused the latter to fall on the ground, would have been sufficient to repel the
attack allegedly initiated by the latter.

But Flores continued shooting Jesus. Considering the number of gunshot wounds sustained by the victim, the Court finds
it difficult to believe that Flores acted to defend himself to preserve his own life. "It has been held in this regard that the
location and presence of several wounds on the body of the victim provide physical evidence that eloquently refutes
allegations of self-defense."37
Facts: On August 15, 1989, on the eve of the barangay fiesta in San Roque, Alaminos, Laguna, certain visitors, Ronnie
de Mesa, Noli de Mesa, Marvin Avenido, and Duran, were drinking at the terrace of the house of Jesus. They started
drinking at 8:30 o’clock in the evening. Jesus, however, joined his visitors only at around 11:00 o’clock after he and his
wife arrived from Sta. Rosa, Laguna, where they tried to settle a problem regarding a vehicular accident involving one of
their children. The drinking at the terrace was ongoing when Flores arrived with an M-16 armalite rifle. 4

Duran testified that Jesus stood up from his seat and met Flores who was heading towards the terrace. After glancing at
the two, who began talking to each other near the terrace, Duran focused his attention back to the table. Suddenly, he
heard several gunshots prompting him to duck under the table. Right after the shooting, he looked around and saw the
bloodied body of Jesus lying on the ground. By then, Flores was no longer in sight. 5

Duran immediately helped board Jesus in an owner-type jeep to be brought to a hospital. Thereafter, Duran, Ronnie de
Mesa and Noli de Mesa went home. Jesus was brought to the hospital by his wife and children. Duran did not, at any time
during the occasion, notice the victim carrying a gun with him. 6

Gerry narrated that he was going in and out of their house before the shooting incident took place, anxiously waiting for
the arrival of his parents from Sta. Rosa, Laguna. His parents were then attending to his problem regarding a vehicular
accident. When they arrived, Gerry had a short conversation with his father, who later joined their visitors at the terrace. 7
Gerry was outside their house when he saw Flores across the street in the company of some members of the CAFGU. He
was on his way back to the house when he saw Flores and his father talking to each other from a distance of about six (6)
meters. Suddenly, Flores shot his father, hitting him on the right shoulder. Flores continued shooting even as Jesus was
already lying flat on the ground. Gerry testified that he felt hurt to have lost his father. 8

Issue: WHETHER THE SANDIGANBAYAN, FIRST DIVISION, GRAVELY ERRED IN NOT GIVING DUE CREDIT TO
PETITIONER’S CLAIM OF SELF-DEFENSE.

[G.R. No. 109614-15. March 29, 1996]


PEOPLE OF THE PHILIPPINES, plaintiff-appellee, vs. ADRONICO GREGORIO and RICARDO
GREGORIO,defendants-appellants.

ARTICLE

ARTICLE 11. JUSTIFYING CIRCUMSTANCES – The following do not incur any criminal liability:

1. Anyone who acts in defense of is person or rights, provided the following circumstances concur:
First. Unlawful aggression;
Second. Reasonable necessity of the means employed to prevent or repel it;
Third. Lack of sufficient provocation on the part of the person defending himself.

According to the court the guiding jurisprudential principle has always been that when an accused invokes the
justifying circumstance of self-defense, the burden of proof is shifted to him to prove the elements of that claim; otherwise,
having admitted the killing, conviction is inescapable. And that he must rely on the strength of his own evidence and not
on the weakness of the prosecution. Having admitted the killing, appellant has to justify his taking of a life by the exacting
standards of the law. For self-defense to prosper, the following requisites must concur: (1) there must be unlawful
aggression by the victim; (2) that the means employed to prevent or repel such aggression were reasonable; and (3) that
there was lack of sufficient provocation on the part of the person defending himself.
If the appellants stabbed Carlos Catorse and Marcelo Lo to defend themselves it certainly defies the reason why
they had to inflict 16 stab wounds and 6 respectively. The location, number and gravity of the wounds inflicted on the
victims belie the appellants contention that they acted in self-defense. The nature and extent of the wounds inflicted on a
victim negate an accused claim of self-defense.

FACTS

Around 8:00 PM of May 7, 1986, Carlos Catorse together with his fifteen year old son Romeo Catorse arrived at
the house of appellant Adronico Gregorio at Sitio Bug-as, Barangay Sta. Cruz, Murcia, Negros Occidental, to attend the
wake of the latters grandson.
Persons attending the wake were requested by appellant Adronico to deposit with him any weapon in their possession for
safekeeping so as to avoid trouble. Complying therewith, Carlos Catorse handed over his samurai, John Villarosa and
Remolito Calalas, their respective knives, to Adronico . Around 1:00 oclock in the morning of May 8, 1986, while the game
of pusoy was still in progress, appellant Ricardo, in a very loud voice, reprimanded Tunggak (son of Adronico) from
peeping at the cards of other players. In response, Tunggak stood up and also in a very loud voice ordered the game
stopped).
Overhearing the incident, Adronico ordered his son downstairs and right there and then scolded and boxed him several
times. While Adronico was severely beating Tunggak, Carlos Catorse approached and begged Adronico from further
hurting his son so as not to put him to shame before the crowd. Carlos was in this act of pacifying the matter between the
father and son when suddenly appellant Ricardo stealthily stabbed Carlos from behind with a samurai (the same samurai
deposited by Carlos to Adronico) and thereafter hacked and stabbed him several times more in different parts of his body:
Right after Carlos fell to the ground, Adronico, for his part, repeatedly hacked the victim with a bolo.
Romeo Catorse, son of Carlos Catorse, ran out of the house. Jovito Nicavera also tried to get out of the house but
Adronico hacked him instead with a bolo hitting his left shoulder. Marcelo Lo tried to help his uncle Jovito but Ricardo, with
the same samurai used against Carlos hacked him on his forearm. Adronico immediately followed and using a bolo
hacked Marcelo on the nape. Although wounded, Marcelo was able to run out of the house but Adronico ran after and
overtook him. Adronico then hacked him again.
Ricardo died during the trial. Adronico on the other hand interposed self-defense to exculpate himself from criminal
liability.
ISSUE
Whether or not the appellant can validly invoke self-defense.

P/CPL. FAUSTO ANDAL v. SANDIGANBAYAN AND PEOPLE OF THE PHILIPPINES


G.R. No. L-60159, 06 November 1989

Introduction (Provisions involved, principles, jurisprudence):

The petitioner failed to prove the defense he had raised. The primordial requisite of self-defense is unlawful aggression.
And for unlawful aggression to be present, there must be a real danger to life or personal safety. In the instant case, there
was no imminent and real danger to the life or limb of the petitioner when he shot the deceased, since the latter had
already been disarmed. As former Chief Justice Aquino states in his book on Criminal Law:

In order to justify self-defense, it is essential that the attack upon defendant be simultaneous with the killing, or
preceded the latter without an appreciable interval of time. (Ferrer, 1 Phil. 56),

xxx xxx xxx

The harm caused by one person to another who offended or caused him injury, sometime after he suffered such
offense or such injury, does not constitute an act of self-defense, but an act of revenge. (Banzuela 31 Phil. 564)

In imposing on the appellant the penalty of just one (1) year of prision correccional, the respondent Court held (which the
Supreme Court affirmed):

xxx xxx xxx

Article 69 of the Revised Penal Code vests discretion to the court in lowering the penalty either by one or two
degrees whenever incomplete justifying circumstance exists in a given case like the case at bar. The laudable
patience of accused in not retaliating despite repeated insults by a subordinate, his length of service in the
government (since 1957), and most important, his obsession to inculcate discipline in his men, to OUR mind,
entitle accused to a two-degree reduction of the penalty prescribed by law. Our attitude is a signal to the men in
uniform that while WE condemn felonious violence WE support efforts to maintain discipline in the service.

Brief facts:

At about 0700pm on 25 September 1980, Petitioner Fausto Andal, then a corporal in the Batangas Integrated National
Police, while on patrol aboard a tricycle driven by Police Pfc. Casiano Quinio, went to the police station at the pier located
at Sta. Clara, Batangas City to check on one of his men, Pfc. Maximo Macaraig, who was stationed there, because the
said Macaraig had failed to report to police headquarters for briefing but Macaraig replied that he did not have to report to
police headquarters since he already had his orders and told the petitioner: "You report, supsup, ka.”

When petitioner Fausto Andal and Quinio went back to the poblacion of Batangas City and parked their vehicle in front of
the Philbanking Building at P. Burgos Street, Batangas City, Macaraig arrived and went straight to the petitioner as he was
furious and demanded why the petitioner had embarrassed him in front of so many people. The petitioner denied the
charge and Quinio told Macaraig that the petitioner did not utter defamatory words against him and asked him to forget
the incident. But still Macaraig challenged the petitioner and fired his gun at the petitioner, hitting the latter in the middle
aspect, lower right knee. The petitioner was able to wrest the gun from Macaraig when they grappled for the possession of
the gun and two (2) successive shots were fired at Macaraig by the petitioner which caused Macaraig’s death.

Brief issue:

Was there an appreciable time lapse between the first aggression, i.e. when deceased shot accused on his knee and the
time accused resorted to force by way of firing the two shots at the deceased? The facts unfolded indicate that there was.
This is what happened after accused had grabbed the gun: (1) He asked deceased, "Why did you fire at me?" (2) He even
turned his head towards his son and instructed him just to stay in the jeep. (3) His son, Domingo Andal, challenged
deceased to a fight "Sportsman like." (4) Deceased moved backward — 2 meters away from accused. (5) Pfc. Quinio
even thought the trouble was over as he started to get his tricycle.

Case no. 25

PEOPLE OF THE PHILIPPINES vs. GERARDO SAZON, alias "INSIK,"


G.R. No. 89684 | September 18, 1990
REGALADO, J.

DOCTRINE / RULING:
Self-Defense; Well-entrenched is the rule that where the accused invokes self-defense, it is incumbent upon him to
prove by clear and convincing evidence that he indeed acted in defense of himself. He must rely on the strength of his
own evidence and not on the weakness of the prosecution. For, even if the prosecution evidence is weak, it could not be
disbelieved after the accused himself had admitted the killing.

It is a statutory and doctrinal requirement that for the justifying circumstance of self-defense, the presence of unlawful
aggression is a condition sine qua non. There can be no self-defense, complete or incomplete, unless the victim has
committed an unlawful aggression against the person defending himself. In the present case, the burden of evidence
having been shifted, we hold that the defense failed to establish the primary element of unlawful aggression on the part of
the victim and, therefore, the plea of self-defense must fail. The narrations of the sequence of events by the accused, and
by the lone alleged eyewitness for the defense, are unconvincing primarily on account of their inherent inconsistency and
conflict with each other.
Qualifying Circumstances; Under such considerations and there being no other evidence to prove that the death of the
victim was the result of meditation, calculation or reflection, evident premeditation cannot be appreciated to qualify the
killing to murder. The circumstances qualifying or aggravating the act must be proved in an evident and incontestable
manner. They must be proved as conclusively as the acts constituting the offense. Thus, for the same reason, the
aggravating circumstance of abuse of superior strength cannot be appreciated in this case. Superior strength may
aggravate or qualify a crime, only if it is clearly shown that there was deliberate intent to take advantage of it. Absence of
any evidence to show that the accused purposely sought to use their superior strength to their advantage in this case, a
finding to that effect by the trial court cannot be sustained.
Conspiracy; The fact that appellant did not inflict the mortal wound upon the deceased is of no moment, since the
existence of conspiracy was satisfactorily shown by the evidence. The coordinated acts of appellant and Altejos of
immediately following the victim and jointly confronting him thereafter reveal a concordance and unity of thought which
resulted in the encounter. The circumstances that after the accused shot the victim in the forearm and, while he and the
victim were grappling for appellant’s gun, Altejos stabbed the victim to death, indicate closeness and coordination of their
action geared towards a common purpose, that is, to kill the victim. Proof of a previous agreement to commit the crime is
not absolutely essential to establish a conspiracy. It is sufficient that the accused be shown to have acted in concert
pursuant to the same objective, as such circumstance is invariably indicative of a conspiratorial agreement.
The rule is that where a conspiracy is proven, a showing as to who inflicted the fatal wound is not required to sustain a
conviction. The act of one in killing the victim becomes the act of all the accused.

FACTS:
Ernesto Romualdez was accosted by appellant near the barangay hall for allegedly circulating the rumor that appellant
and his companions were engaged in stealing. Upon confrontation, appellant boxed Romualdez which caused the latter to
fall. Wilfredo Longno, who was then present at the scene, approached and helped the fallen Romualdez and pushed
appellant away. This apparently angered appellant who, in his native dialect said, “Andam ka lang Inday kay patyon ta
guid,” (“Watch out Inday for I will kill you”) to which Longno retorted, “Just do it.”

Two days later, appellant and his cousin, Cornelio Altejos, were drinking softdrinks at the store of Gloria Aposaga when
Longno passed by. Thereupon, appellant and Altejos left their softdrinks half-consumed and followed Longno who
eventually reached the bench near the public faucet where a group of guys were sitting and joined the group in their
conversation. Shortly thereafter, appellant and Altejos arrived and appellant accosted and pointed a gun at Longno,
saying, “Maano ka?” (“What are you going to do?”). Longno then faced appellant and said, “Brod, tiruha lang.” (“Brod, just
shoot.”)

Apparently irked by the response, appellant fired the gun, hitting Longno in the left forearm. Dullete, Canoso and Ramos
then scampered for safety as appellant and the wounded Longno grappled for the gun. It was while the two were thus
struggling that Altejos stabbed Longno in the chest, after which both appellant and Altejos ran away.

Appellant’s version of the incident, however, differs. He admits having shot Longno but pleads self-defense. He claims that
Altejos for help to have a .22 caliber revolver repaired and appellant was taking the revolver to a policeman friend of his.
On their way, appellant saw Longno from a distance. Upon his approach, Longno allegedly said, “Insik, I heard that you
are not afraid of me. Maybe you want to be taught a lesson.”

Appellant claims that the deceased had a revolver tucked in his waist and was about to draw the same. He, therefore,
parried the gun but it fired hitting one of appellant’s left fingers which was later amputated. It was then that appellant
pulled out his gun and shot Longno in the forearm. Appellant and Longno afterwards grappled for the gun. Altejos
allegedly tried to separate appellant and Longno but he was brushed aside by the latter. In the course of their struggle,
Altejos then shouted to appellant, “I stabbed Inday, run,” and so he and Altejos ran away.

ISSUE/S:
1. Did the accused act in self-defense?
2. Whether or not qualifying circumstances are present?
3. Is there a conspiracy between the accused and Altejos?

Note:
Insofar as Cornelio Altejos is concerned, however, the trial court never acquired jurisdiction over him and he can neither
be convicted nor exculpated herein. References in this judgment to him are, therefore, obiter and with no binding effect on
him.

PEOPLE v. ALCONGA
Facts:
As the deceased Barion and the defendant Alconga were playing black jack, the former became outraged and expressed
his rage at the latter for his cheating tactics. Two (2) days later, making good the threat uttered after the game, Barion
approached the defendant and swung his pingahan, a bamboo stick. A fight ensued afterwards, upon which Alconga
gained the upper hand by firing his revolver at Barion and eventually inflicting a mortal bolo blow, slashing the latter’s
cranium. The argument of self defense was raised by Alconga.
Issues:
1. Whether or not the Alconga’s plea of self defense can be sustained
2. Whether or not Barion’s provocation is considered a mitigating circumstance
Ruling:
1. NO.
There were two (2) stages in the fight- the first wherein the deceased was the aggressor and the second wherein
there is no more aggression, since Barion was already injured and fleeing. Under such circumstances, Alconga’s plea
of self-defense (as a justifying circumstance) during the second stage of the fight cannot be sustained. There can be
no defense where there is no aggression, given that the aggression must be deemed to have ceased upon the flight
of the deceased during the first stage. A fleeing man is not dangerous to the one from whom he flees.
2. NO.
Section 4, Article 13 of the Revised Penal Code requires that there be sufficient provocation or threat on the part
of the offended party immediately preceding the act. Sufficiency, as defined by Justice Albert, means “that it should be
proportionate to the act committed and adequate to stir one to its commission.” It would seem self-evident that
Alconga could never have succeeded in showing that Barion’s aggression was proportionate to his killing his already
defeated adversary. Also, it is noteworthy to mention that at Alconga, as a guard by profession, was the holder of
more deadly weapons and has superior fighting ability than Barion.

THE PEOPLE OF THE PHILIPPINES vs. BENJAMIN GONDAYAO, alias "BEN", ET AL.
Article 11. Justifying Circumstances- 1. Anyone who acts in defense of his person or rights, provided that the following
circumstances concur: First. Unlawful aggression; Second. Reasonable necessity of the means employed to prevent or
repel it; Third. Lack of sufficient provocation on the part of the person defending himself.
Article 13. Mitigating Circumstance- 1. Those mentioned in the preceding chapter, when all the requisites necessary to
justify the act or to exempt from criminal liability in the respective cases are not attendant.
Article 14. Aggravating Circumstance- 16. That the act be committed with treachery (alevosia).
There is treachery when the offender commits any of the crimes against the person, employing means, methods, or forms
in the execution thereof which tend directly and specially to insure its execution, without risk to himself arising from the
defense which the offended party might make.
No unlawful aggression when there is an agreement to fight. The challenge to a fight must be accepted. The reason for
the rule is that each of the protagonists is at once assailant and assaulted and neither can invoke the right to self-defense
because aggression which is incident in the fight is bound to arise from one or the other of the combatants.
Unlawful aggression must come, directly or indirectly, from the person who was subsequently attacked by the accused.
RULING: Benjamin’s story is manifestly artificious and unworthy of credence. It should be noted that, according to
Benjamin, he held the dagger, even before they fell from the "papag"; that the dagger was still in his hand when Piol
allegedly struck his face with a stone; and that he (Benjamin), in turn, took another piece of stone, and hit Piol with it. This
would have been impossible, however, unless Benjamin first released the dagger, which he then held; but, We cannot
believe that he, or anybody for that matter, would have done so under the circumstances.
Again, when Piol allegedly squeezed the neck of Benjamin, the latter threw the stone away and picked up the dagger
once more. He would thus have Us believe that, in order to get the stone with which he claimed to have hit Piol on the
head, he put the dagger in a convenient place from which, at the opportune moment, he got it back to inflict the second
stab wound. The context of Benjamin's story does not convey the idea that he had such a control of the situation as to be
able to choose the place where he would put the dagger and the time he would retrieve it.
But, this is not all. Instead of stabbing Piol on the stomach, for, by this time, he was again lying down on his back —
according to the defense — with Benjamin on top of him, he (Benjamin) thrust the dagger, with his left hand, into the back
of Piol, causing therein another stab wound almost at right angle with his body, like the first. Just why, being in the
precarious condition he depicted himself, Benjamin chose to stab Piol in such an awkward, inconvenient and unbelievable
manner, the defense has not even tried to explain. Regardless of the foregoing, an injury inflicted in this fashion on Piol's
back, which was allegedly pressed against the ground, would have necessarily been much more slanting than the first,
instead of being almost perpendicular to the body. The fact of the matter — and this has been established by the
testimony of the Chief of Police, whose impartiality and veracity are not contested — is that Piol was then lying down, not
on his back, but on his stomach with Benjamin on top of him. This explains why and how he (Benjamin) managed to stab
Piol on the back. It, likewise, shows that Piol could not have struck Benjamin on the face with a stone, much less
squeezed his neck.
Benjamin stabbed Piol twice from behind, after disarming him. Considering, moreover, that Benjamin had provoked the
incident, by hurling uncomplimentary remarks at his political opponents, one of whom was Piol; that such remarks led to
an altercation with Piol, in consequence of which, stones were thrown at him, hitting him on the head; that when, owing to
the impact of said stone, which could have rendered him groggy, and the lacerated injuries thus sustained by him, Piol
prepared himself to fight by drawing out his dagger, Benjamin accepted the challenge resulting from this act, by "rushing"
to his encounter and grappling with him; and that, accordingly, Benjamin cannot be given the benefit of either complete or
incomplete self-defense. Although Piol was stabbed from behind, Benjamin did not act with treachery, for this was merely
an incident of their struggle, which had begun with both contenders facing each other, each prepared for the fight that
ensued.
The crime committed by Benjamin Gondayao is, therefore, that of homicide, and no modifying circumstance having
attended its perpetration, the penalty therefor (reclusion temporal) should be imposed in its medium period. Pursuant to
the Indeterminate Sentence Law, he should, accordingly, be sentenced to an indeterminate penalty ranging from 8 years
and 1 day of prison mayor, as minimum, to 14 years, 8 months and 1 day of reclusion temporal as maximum, with the
corresponding accessory penalties.
FACTS: Appeal, taken by defendants Benjamin Gondayao and Anoy Gondayao from a decision convicting them of the
crime of murder of Orlando Piol, qualified by treachery.
After consuming a bottle of gin, Benjamin, repeatedly cursed the "Nacionalistas" for not offering a drink despite their
victory in the polls; that, as Piol who was nearby, replied by cursing the losers in the elections, an exchange of unfriendly
utterances followed; that, eventually, the group headed by Benjamin picked up stones and threw them at Piol, who was hit
several times on the head; that, as the same began to bleed, Piol drew out a dagger and approached Benjamin, who,
forthwith, embraced him; that, grappling with each other, they both fell down.
After grappling with Benjamin Gondayao, Orlando Piol appeared to have — in addition to several lacerations on the head
— two (2) stab wounds on the back, 4 to 4-1/2 inches deep, in consequence of which he died due to a massive internal
hemorrhage. Immediately after the occurrence, Benjamin assumed full responsibility for said injuries, which he claimed to
have inflicted in self-defense.
Benjamin testified that his remark, about the failure to give a blow-out on the part of the winner in the elections, was
addressed to his nephew, Rudy Natividad; that, when Piol resented said remark, Benjamin replied that the same was not
aimed at him, and that, perhaps, he (Piol) is a "nacionalista".
ISSUE: Whether or not the accused acted in self-defense.
G.R. No. L-28129 October 31, 1969
ELIAS VALCORZA, petitioner,
vs.
PEOPLE OF THE PHILIPPINES, respondent.

RULING
The petitioner is acquitted.

ART. 11. Justifying circumstances. - The following do not incur any criminal liability
Paragraph 5. Any person who acts in the fulfillment of a duty or in the lawful exercise of a right or office.

The facts and circumstances constrain us to hold that the act thus performed by petitioner, which unfortunately resulted in
the death of the escaping detainee, was committed in the performance of his official duty and was more or less necessary
to prevent the escaping prisoner from successfully eluding the officers of the law. To hold him guilty of homicide may have
the effect of demoralizing police officers discharging official functions identical or similar to those in the performance of
which petitioner was engaged at the time he fired at the deceased Pimentel, with the result that thereafter We would have
half-hearted and dispirited efforts on their part to comply with such official duty. This of course, would be to the great
detriment of public interest.

FACTS:
The deceased, Roberto Pimentel was a detention prisoner who escaped. While in search for the escaped prisoner, Sgt.
Daiton saw a person approaching slowly under the bridge and he ordered him to halt. The latter instead of doing so,
jumped down into the creek spanned by the bridge. Roberto Pimentel emerged suddenly from the bushes and lunged at
the appellant Valcorza, hitting him with a stone and causing him to fall to the ground. Appellant Valcorza regained his
composure and immediately chased the deceased, as the deceased did not heed his order to stop, appellant fired four
times into the air, and a fifth shot at Pimentel as the latter was in an act of again jumping down into another part of the
creek. The members of the patrol team went down into the water to locate Pimentel and they saw him floating, with a
wound on his back. Elias Valcorza surrendered himself and his firearm to the Chief of Police

ISSUE:
Whether or not peace officer Valcorza was justified in shooting the deceased

PEOPLE OF THE PHILIPPINES vs. AVELINO MANANSALA, JR. and JOSE MANANSALA
G.R. No. L-23514, February 17, 1970

Article 17 of the RPC; Article 18 of the RPC states that Accomplices are those persons who, not being included in Art.17,
cooperate in the execution of the offense by previous or simultaneous acts. In this case, Jose Manansala was found guilty
as co-principal on the ground that there was concert of action between him and his nephew. The evidence does not justify
this finding beyond reasonable doubt. There is no showing that the killing was agreed upon between them beforehand. No
motive for it has been shown other than the provocation given by the deceased; and such motive was true only insofar as
Avelino was concerned. The circumstances indicate that if Jose embraced Rodrigo and rendered him helpless, it was to
stop him from further hitting Avelino with his fists. However, Jose is not entirely free from liability, for it has been
established that even after the first knife thrust had been delivered he did not try to stop Avelino, either by word or overt
act. Instead Jose continued to hold Rodrigo, even forced him down on the bamboo bed with Avelino still pressing the
attack. Withal it cannot be said that Jose's cooperation was such that without it the offense would not have been
accomplished. But although not indispensable, it was a contributing factor. If Jose's initial intent was free from guilt, it
became tainted after he saw the first knife thrust delivered. The thirteen wounds must have taken an appreciable interval
of time to inflict, and Jose's cooperation facilitated their infliction. He must therefore be held liable as an accomplice.

Facts: Avelino was eating his lunch inside one of the eateries dotting the market site, when Rodrigo Aringo alias Diego, a
baggage boy in the same market, approached him and demanded his fee for having carried Avelino's baggage. When
Avelino said he was willing to pay for the services rendered at noon, but not for those rendered earlier in the morning,
Rodrigo brusquely brushed Avelino's hand aside and instantly gave him a fist blow in the face. A quarrel between them
ensued and Jose Manansala, Avelino's uncle, noticed the commotion and so he went there and had Rodrigo in a tight
embrace. While in that position, Avelino stabbed him with a balisong, or Batangas knife thirteen times which resulted to
Rodrigo's death. Avelino and his uncle, Jose were charged with murder and Jose was considered as a co-principal by the
CFI.

Issue: Whether or not Jose Manansala was correctly considered as a co-principal by the CFI?

Case #32.
People of the Philippines vs Jose Encomienda
GR No L-26750; August 18, 1972

Article 11, Revised Penal Code/ Legitimate Self-Defense, elements: (1) unlawful aggression on part of the victim; (2)
reasonable necessity of the means, employed to prevent or repel the attack; and (3) lack of sufficient provocation on the
part of the person defending himself
 Illegal Aggression, what constitutes: Illegal aggression is equivalent to assault or at least threatened assault of
immediate and imminent kind.

 Reasonable Necessity of the Means Employed, what constitutes: Reasonable necessity of the means
employed does not imply material commensurability between the means of attack and defense. When the law
requires is rational equivalence, in the consideration of which will enter as principal factors the emergency, the
imminent danger to which the person attacked is exposed and the instinct, more than the reason, that moves or
impels the defense, and the proportionateness thereof does not depend upon the harm done, but rests upon the
imminent danger of such injury.

Facts: Accused was charged with the crime of murder aggravated by recidivism. He interposed the lawful self-defense as
per Article 11 of the Revised Penal Code.
Issue: Whether or not the accused satisfied the elements of lawful self-defense?
Held: Yes, the accused sufficiently proved the elements for lawful self-defense. Elements are: (1) unlawful aggression on
part of the victim; (2) reasonable necessity of the means, employed to prevent or repel the attack; and (3) lack of sufficient
provocation on the part of the person defending himself.
It was the deceased who drew his gun towards the accused. Not having enough time to think rationally how to
deal with the situation, the accused’s act of hacking the left arm of deceased is justified due to immediate danger to his life
as the former was about to grab gun from his free left hand. Provocation came from the deceased (not the accused) as he
ordered the accused to vacate land and drew his gun when he did not like the response of the former.
People vs Sotelo
55 PHIL 403
December 13, 1930

Article 13 of the RPC: Incomplete Self Defense

Facts:
The Sotelo brothers, namely, Constante, Dominador, and Vicente, were prosecuted in the Court of First Instance of Ilocos
Sur for the crime of homicide. The said accused conspired to attack Ignacio Cambaliza and inflicted a mortal wound upon
him which he died in a few minutes afterwards.

Issue:
Whether or not the accused Constante can invoke self defense?

Ruling:
The selfdefense invoked by the accused was incomplete, for, although the appellant was unlawfully attacked by the
deceased and compelled to employ reasonable means to defend himself, he is responsible for provoking the attack.

PEOPLE vs. HERNANDEZ


GR NO. L-6025, MAY 30 1964
Labrador, J.:

Article 136. Conspiracy and proposal to commit rebellion or insurrection

FACTS:

About March 15, 1945, Amado Hernandez and other appellants were accused of conspiring, confederating and
cooperating with each other, as well as with the thirty-one (31) defendants charged in the criminal cases of the Court of
First Instance of Manila. They were accused of being members of PKP Community Party of the Philippines which was
actively engaged in an armed rebellion against the government of the Philippines. With the party of HUKBALAHAP
(Hukbo ng Bayan Laban sa mga Hapon), they committed the crime of rebellion causing murder, pillage, looting plunder,
etc., enumerated in 13 attacks on government forces or civilians by HUKS.

ISSUE: Does his or anyone’s membership in the communist party per se render Hernandez or any Communist guilty of
conspiracy to commit rebellion under the provisions of Article 136 of the RPC?

HELD:

No. The advocacy of Communism or Communistic theory and principle is not to be considered as a criminal act of
conspiracy unless transformed or converted into an advocacy of action. In the very nature of things, mere advocacy of a
theory or principle is insufficient unless the communist advocates action, immediate and positive, the actual agreement to
start an uprising or rebellion or an agreement forged to use force and violence in an uprising of the working class to
overthrow constituted authority and seize the reins of Government itself. Unless action is actually advocated or intended
or contemplated, the Communist is a mere theorist, merely holding belief in the supremacy of the proletariat a Communist
does not yet advocate the seizing of the reins of Government by it. As a theorist the Communist is not yet actually
considered as engaging in the criminal field subject to punishment. Only when the Communist advocates action and
actual uprising, war or otherwise, does he become guilty of conspiracy to commit rebellion.

The most important activity of appellant Hernandez appears to be the propagation of improvement of conditions of labor
through his organization, the CLO. While the CLO of which he is the founder and active president, has communistic
tendencies, its activity refers to the strengthening of the unity and cooperation between labor elements and preparing
them for struggle; they are not yet indoctrinated in the need of an actual war with or against Capitalism. The appellant was
a politician and a labor leader and it is not unreasonable to suspect that his labor activities especially in connection with
the CLO and other trade unions, were impelled and fostered by the desire to secure the labor vote to support his political
ambitions. It is doubtful whether his desire to foster the labor union of which he was the head was impelled by an actual
desire to advance the cause of Communism, not merely to advance his political aspirations.

Insofar as the appellant's alleged activities as a Communist are concerned, We have not found, nor has any particular act
on his part been pointed to Us, which would indicate that he had advocated action or the use of force in securing the ends
of Communism. True it is, he had friends among the leaders of the Communist Party, and especially the heads of the
rebellion, but this notwithstanding, evidence is wanting to show that he ever attended their meetings, or collaborated and
conspired with said leaders in planning and encouraging the acts of rebellion, or advancing the cause thereof. Insofar as
the furnishing of the mimeograph machine and clothes is concerned, it appears that he acted merely as an intermediary,
who passed said machine and clothes on to others. It does not appear that he himself furnished funds or material help of
his own to the members of the rebellion or to the forces of the rebellion in the field.

But the very act or conduct of his in refusing to go underground, in spite of the apparent desire of the chief of the rebellion,
is clear proof of his non-participation in the conspiracy to engage in or to foster the rebellion or the uprising.
G.R. No. L-4445 February 28, 1955
THE PEOPLE OF THE PHILIPPINES, plaintiff-appellee,
vs.
MANUEL BERONILLA, FILIPINO VELASCO, POLICARPIO PACULDO, and JACINTO ADRIATICO, defendants-
appellants
FACTS:
Arsenio Borjal was the elected mayor of La Paz, Abra, at the outbreak of war, and continued to serve as Mayor during the
Japanese occupation, until March 10, 1943, when he moved to Bangued because of an attempt upon his life by unknown
persons. On December 18, 1944, appellant Manuel Beronilla was appointed Military Mayor of La Paz by Lt. Col. R. H.
Arnold, regimental commander of the 15th Infantry, Philippine Army, operating as a guerrilla unit in the province of Abra.

Sometime in March, 1945, while the operations for the liberation of the province of Abra were in progress, Arsenio Borjal
returned to La Paz with his family in order to escape the bombing of Bangued. Beronilla, pursuant to his instructions,
placed Borjal under custody and asked the residents of La Paz to file complaints against him. In no time, charges of
espionage, aiding the enemy, and abuse of authority were filed against Borjal.

The trial lasted 19 days up to April 10, 1945; the jury found Borjal guilty on all accounts and imposed upon him instruction
from his superiors. And on the night of the same day, April 18, 1945, Beronilla ordered the execution of Borjal. Jacinto
Adriatico acted as executioner and Antonio Palope as grave digger.

Two years thereafter, Manuel Beronilla as military mayor, Policarpio Paculdo as Clerk of the jury, Felix Alverne and Juan
Balmaceda as prosecutors, Jesus Labuguen, Delfin Labuguen, Filemon Labuguen, Servillano Afos, Andres Afos,
Benjamin Adriatico, Juanito Casel, Santiago Casel, Mariano Ajel, Felix Murphy, Benjamin Abella, and Pedro Turqueza as
members of the jury, Jacinto Adriatico as executioner, Severo Afos as grave digger, and Father Filipino Velasco as an
alleged conspirator, were indicted in the Court of First Instance of Abra for murder, for allegedly conspiring and
confederating in the execution of Arsenio Borjal.

ISSUE:
whether or not this message, originally sent to Arnold's quarters in San Esteban, Ilocos Sur, was relayed by the latter to
appellant Beronilla in La Paz, Abra, on the morning of April 18, 1945, together with the package of records of Borjal's trial
that was admittedly returned to and received by Beronilla on that date, after review thereof by Arnold (Exhibit 8-8-a).
RULING:

We have carefully examined the evidence on this important issue, and find no satisfactory proof that Beronilla did actually
receive the radiogram Exhibit H or any copy thereof. The accused roundly denied it.

. In addition to Balmaceda was contradicted by Bayken, another prosecution witness, as to the hatching of the alleged
conspiracy to kill Borjal. Balmaceda claimed that the accused-appellants decided to kill Borjal in the early evening of April
18, while Bayken testified that the agreement was made about ten o'clock in the morning, shortly after the accused had
denied Borjal's petition to be allowed to hear mass.
Our conclusion is that Lt. Col. Arnold, for some reason that can not now be ascertained, failed to transmit the Volckmann
message to Beronilla. And this being so, the charge of criminal conspiracy to do away with Borjal must be rejected,
because the accused had no need to conspire against a man who was, to their knowledge, duly sentenced to death.

decided that the concurrence of personal hatred and collaboration with the enemy as motives for a liquidation does not
operate to exclude the case from the benefits of the Amnesty claimed by appellants, since then "it may not be held that
the manslaughter stemmed from purely personal motives" Actually, the conduct of the appellants does not dispose that
these appellants were impelled by malice.
The lower Court, after finding that the late Arsenio Borjal had really committed treasonable acts, (causing soldiers and
civilians to be tortured, and hidden American officers to be captured by the Japanese) expressly declared that "the Court
is convinced that it was not for political or personal reason that the accused decided to kill Arsenio Borjal"

Actus non facit reum nisi mens si rea.


To constitute a crime, the act must, except in certain crimes made such by statute, be accompanied by a criminal
intent, or by such negligence or indifference to duty or to consequence, as, in law, is equivalent to criminal intent.
The maxim is, actus non facit reum, nisi mens rea-a crime is not committed if the minds of the person performing
the act complained of be innocent. (U. S. vs. Catolico, 18 Phil., 507).

G.R. No. 137347 March 4, 2004


PEOPLE OF THE PHILIPPINES, appellee,
vs.
PO3 FERDINAND FALLORINA Y FERNANDO, appellant.

Article 248 of the Revised Penal Code, the penalty for murder is reclusion perpetua to death. Since there is no
modifying circumstance in the commission of the crime, the appellant should be sentenced to suffer the penalty
of reclusion perpetua, conformably to Article 63 of the Revised Penal Code.

FACTS:
The accused PO3 Ferdinand Fallorina y Fernando, with intent to kill, by means of treachery and taking advantage of
superior strength, did then and there, wilfully, unlawfully and feloniously attack, assault and employ personal violence
upon the person of VINCENT JOROJORO, JR. y MORADAS, a minor, eleven (11) years of age, by then and there,
shooting him with a gun, hitting him on the head, thereby inflicting upon him serious and mortal wound which was the
direct and immediate cause of his death, to the damage and prejudice of the heirs of the said offended party.
The trial court rendered judgment convicting the appellant of murder, qualified by treachery and aggravated by abuse of
public position. The trial court did not appreciate in favor of the appellant the mitigating circumstance of voluntary
surrender.
ISSUE:
Whether or not the appellant is exempt from criminal liability.

RULING:
The appellant was burdened to prove, with clear and convincing evidence, his affirmative defense that the victim's death
was caused by his gun accidentally going off, the bullet hitting the victim without his fault or intention of causing it; hence,
is exempt from criminal liability under Article 12, paragraph 4 of the Revised Penal Code which reads –
The following are exempt from criminal liability:

4. Any person who, while performing a lawful act with due care, causes an injury by mere accident without fault or
intention of causing it.
The basis for the exemption is the complete absence of intent and negligence on the part of the accused. For the accused
to be guilty of a felony, it must be committed either with criminal intent or with fault or negligence. 33
The elements of this exempting circumstance are (1) a person is performing a lawful act; (2) with due care; (3) he causes
an injury to another by mere accident; and (4) without any fault or intention of causing it. 34 An accident is an occurrence
that "happens outside the sway of our will, and although it comes about through some act of our will, lies beyond the
bounds of humanly foreseeable consequences." If the consequences are plainly foreseeable, it will be a case of
negligence.
The appellant committed murder under Article 248 of the Revised Penal Code qualified by treachery.
Under Article 248 of the Revised Penal Code, the penalty for murder is reclusion perpetua to death. Since there is no
modifying circumstance in the commission of the crime, the appellant should be sentenced to suffer the penalty
of reclusion perpetua, conformably to Article 63 of the Revised Penal Code.
People VS. Ong

Intro:
All contrary to law with the following generic aggravating circumstances:
(a) Evident premeditations;
(b) Grave abuse of confidence;
(c) Nighttime;
(d) Use of an motor vehicle ;
(e) Use of uperior strenght;
(f) Cruelty.

Kidanapping with Murder as defined under Article 248 of the Revised Penal Code, in relation to Article 267

The aggravating circumstance of abuse of superior strength is absorbed in treachery. The aggravating circumstance of
nighttime (nocturnidad) cannot be absorbed in treachery because in this crimes," treachery arose from the defenseless
position of Chua when he was killed, while nighttime was purposely sought by the accused to facilitate immunity in the
commission of the crime. The aggravating circumstance of uninhabited place (despoblado) is also present, due to the
deliberate selection of an isolated place (Barrio Makatipo Novaliches, Caloocan City) for killing and burying the victim

The aggravating circumstance of use of motor vehicle in the commission of the crimes," can be considered present
because the Biscayne car of Ong was used to trail the victim's car and to facilitate the commission of the crimes," and the
escape of the accused.

Evident premeditation attended the commission of the crimes, because the accused meditated, planned, and tenaciously
persisted in the accomplishment of the crime.

Accused Ong was given the mitigating circumstances of plea of guilty and one analogous to passion and obfuscation"
because Chua previously threatened Ong for non-payment of debt arising from gambling, causing Ong humiliation and
shame.
"Conspiracy, connivance and unity of purpose and intention among the accused were present throughout in the execution
of this crime. The four participated in the planning and execution of the crimes," and were at the scene in all its stages.
They cannot escape the consequences of any of their acts even if they deviated in some detail from what they originally
thought of. Conspiracy implies concert of design and not participation in every detail of the execution. Thus, treachery
should be considered against all persons participating or cooperating in the perpetration of the crime."

Brief Facts:
That on or about April 23 to April 24, 1971, inclusive, in the municipality of Parañaque, province of Rizal, Philippines, and
within the jurisdiction of this Honorable court, the above named accused, being then private individuals, conspiring and
confederating together and mutually helping one another, did then and there wilfully, unlawfully and with treachery and
known premeditation and for the purpose of killing one Henry Chua and thereafter extorting money from his family through
the use of a ransom note, kidnapped and carried away said Henry Chua, initially by means of friendly gestures and later
through the use of force, in an automobile, and later after having taken him to an uninhabited place in Caloocan City, with
the use of force detained him (Henry Chua) and killed him in the following manner, to wit: The accused after gagging and
tying up Henry Chua and repeatedly threatening him with death, assured him that if he would write and sign a ransom
note for the payment by his family of the sum of $50,000.00 (US), he would not be killed and would be released upon
receipt of the ransom note, he was again gagged and tied up by the accused, and thereafter stabbed in the abdominal
region, several times with an ice-pick, inflicting upon him (Henry Chua) mortal wounds on his vital organs, which directly
caused his death.

Brief Issue:

The only issue in this case, therefore, is whether or not the accused Ambrosio voluntarily participated in the commission of
the crime.

Case #41

PEOPLE OF THE PHILIPPINES, plaintiff-appellee, vs. GEORGE CORTES y ORTEGA, accused-appellant. [G.R. No.
137050. July 11, 2001]

*in this case the court took little discussions on the application of the following aggravating circumstances as alleged in the
complaint.

I. Evident Premeditation.
In the aggravating circumstance of evident premeditation, the prosecution must prove the following elements:
(a) the time when the accused determined to commit the crime,
(b) an act manifestly indicating that the accused clung to that determination, and
(c) a lapse of time between the determination and the execution sufficient to allow the accused to reflect upon the
consequences of the act
In the case at bar, the prosecution fails to prove this aggravating Circumstance
II. Cruelty
For cruelty to be appreciated against the accused, it must be shown that the accused, for his pleasure and satisfaction,
caused the victim to suffer slowly and painfully as he inflicted on him unnecessary physical and moral pain.
Rationale: The crime is aggravated because by deliberately increasing the suffering of the victim the offender denotes
sadism and consequently a marked degree of malice and perversity.
In the case at bar, The mere fact of inflicting various successive wounds upon a person in order to cause his death, no
appreciable time intervening between the infliction of one (1) wound and that of another to show that he had wanted to
prolong the suffering of his victim, is not sufficient for taking this aggravating circumstance into consideration.

III. Nighttime
Night-time becomes an aggravating circumstance only when:
(1) it is specially sought by the offender;
(2) the offender takes advantage of it; or
(3) it facilitates the commission of the crime by insuring the offender's immunity from identification or capture.
In the case at bar, no evidence suggests that accused purposely sought the cover of darkness to perpetrate the crime, or
to conceal his identity.

IV. Abuse of Superior Strength


Abuse of superior strength is absorbed in treachery, so that it cannot be appreciated separately as another aggravating
circumstance. Here, treachery qualified the offense to murder.

V. Disregard of Sex
As to the aggravating circumstance of disregard of sex, the same could not be considered as it was not shown that
accused deliberately intended to offend or insult the sex of the victim, or showed manifest disrespect for her womanhood.
In the case at bar, the accused mistook the victim for a man

VI. Intoxication
Ordinarily, intoxication may be considered either aggravating or mitigating, depending upon the circumstances attending
the commission of the crime.
Intoxication has the effect of decreasing the penalty, if it is not habitual or subsequent to the plan to commit the
contemplated crime; on the other hand, when it is habitual or intentional, it is considered an aggravating circumstance.
A person pleading intoxication to mitigate penalty must present proof of having taken a quantity of alcoholic beverage prior
to the commission of the crime, sufficient to produce the effect of obfuscating reason. At the same time, that person must
show proof of not being a habitual drinker and not taking the alcoholic drink with the intention to reinforce his resolve to
commit the crime
FACTS:
a.i.1. an Information for murder (violation of Article 248 of the Revised Penal Code) against accused On George Cortes
y Ortega, for the murder of Edlyn S. Gamboa, a 16 year old girl
a.i.2. Accused admitted that he stabbed Edlyn.
a.i.3. He entered a plea of guilty In virtue of his plea of guilty, the trial court proceeded to satisfy itself of the
voluntariness of the plea by propounding questions to the accused to find out if he understood his plea and the legal
consequence thereof.
a.i.4. The prosecution alleged that the aggravating circumstances of evident premeditation, cruelty, nighttime, abuse of
superior strength, disrespect to sex, and intoxication were present in the commission of the crime.
a.i.5. The accused, on the other hand, raised the attendance of the mitigating circumstances of voluntary surrender,
plea of guilty, mistaken identity and the alternative mitigating circumstance of intoxication.
a.i.6. the trial court after considering the aggravating and mitigating circumstances attendant found the existence of the
aggravating circumstances and appreciated only the mitigating circumstance of plea of guilty that was offset by one of the
aggravating circumstances. Hence, this review

ISSUES:
Whether the trial court erred in finding that the aggravating circumstances of evident premeditation, cruelty, nighttime,
abuse of superior strength, sex and intoxication attended the commission of the crime charged.

HELD:
The Solicitor General agrees with the accused that the only aggravating circumstance present was treachery which
qualified the killing to murder and that there were two mitigating circumstances of plea of guilty and intoxication, not
habitual. The penalty shall be reclusion perpetua, not death, in accordance with Article 63 in relation to Article 248 of the
Revised Penal Code, as amended by Republic Act No. 6759

CASE No. 42
People vs Regala
G.R. No. 130508 April 5, 2000

FACTS:
On the night of September 11, 1995, at Barangay Bangon in Aroroy, Masbate, then 16-year old victim Nerissa Tagala and
her grandmother, Counselo Arevalo, were sleeping, when appellant Armando Regala and his two other companions
entered the former’s house.

Appellant and his companions entered the house through the kitchen and went to the room of the victims and poked at 8-
inch gun on them, one after the other, and hogtied both of them. Armando raped Nerissa in bed while her grandmother
was hogtied on the floor. Later, she saw her grandmother’s aparador being opened where two rings, two wrist watches,
and money were taken from the aparador. After raping her in bed, Nerissa saw accused-appellant counting the money
taken from the aparador. Thereafter, she was brought to the kitchen, still hogtied and was raped again by the accused.

He was convicted in the lower court but accused-appellant appealed his criminal case at the Regional Trial Court in
Masbate. He questioned the sufficiency of the prosecution’s evidence in identifying him as one of the perpetrators of the
crime charged. And based on medico-legal, Dr. Conchita Ulanday, a health officer of Aroroy, testified herself that the
complaining witness “either” voluntarily submitted to a sexual act or was forced into one.

ISSUE:
(a) Whether additional rape committed in a crime of robbery be considered as an aggravating circumstance?

HELD:
On cross-examination, both Nerissa Tagala and Consuelo Arevalo, separately testified that they saw the face of Regala,
despite of no electricity at the commission of the crime, because he used a flashlight and took off the mask he was
wearing, and thus, they remembered him wearing an earring of his left ear, which he was still wearing at the time of the
police line-up inside the police station.

The trial court held that contradiction referred to a minor detail, cannot detract from the fact, that both Nerissa and
Consuelo positively identified the accused-appellant. As correctly pointed out by the appellee, the victim was a 16-year old
barrio lass, not exposed to the ways of the world and was not shown to have any ill-motive to falsely implicate accused-
appellant, who was a stranger. Hence, Dr. Ulanday’s testimony does not support the contention of accused-appellant that
the victim voluntarily submitted to sexual advances of Regala.

The crime of robbery with rape was committed in 1995 when RA 7659 was already in force. Under Article 294 of the
Revised Penal Code as amended, now provides, under paragraph 1 thereof: (1) The penalty of reclusion perpetua to
death, when for any reason of or on occasion of the robbery, the crime of homicide shall have been committed, or when
the robbery shall have been accompanied by rape or intentional mutilation or arson.

In this case, the additional rape committed by herein accused-appellant should not be considered as aggravating. The
penalty of reclusion perpetua imposed by the trial court is proper. The judgment convicting Armando Regala y Abriol guilty
beyond reasonable doubt of the crime of Robbery with Rape, where the victim is entitled to an additional award of
P50,000.00 as civil indemnity.

PEOPLE vs BERDIDA
GR No. L-20183; June 30, 1966

DOCTRINE:
Rule of aggravating circumstances with regards to penalty – the presence of one generic aggravating circumstance, apart
from the qualifying circumstance of treachery, suffices to fix the penalty for murder at the extreme punishment of death.
Nighttime – from the facts and evidence of record in this case, appellants took advantage of nighttime in committing the
felonies charged for they had evidently chosen to execute their victims under cover of darkness, at the dead of night,
when the neighborhood was asleep.
Exception to the rule of absorbing Nighttime in Treachery – in as much as the treachery consisted in the fact that the
victims' hands were tied at the time they were beaten, the circumstance of nighttime is not absorbed in treachery, but can
be perceived distinctly therefrom, since the treachery rests upon an independent factual basis. A special case therefore is
present to which the rule that nighttime is absorbed in treachery does not apply.
Evident Premeditation – from the time the accused abducted the victims up to the time one of the latter lost
consciousness and the other died, sufficient time had lapsed for the accused to meditate and reflect on the consequences
of their act.

FACTS:
At around 10:00pm, Antonio, Federico, Virgilio and Pedrito had just left a store and were on their way to each of their
homes, when a group of men armed with bolos stopped them and introduced themselves as policemen, saying that the
four are accused of killing someone and that they should go with them. Antonio and Federico held their hands up while
Virgilio and Pedrito could ran away. Antonio and Federico were taken to a rail track where their hands were tied. They
were then further taken to a pier in North Harbor where they were met by more armed men. The group of men beat
Antonio and Federico until Antonio lost consciousness at around 1:00am of the following day. The police, together with
Antonio's sister, found Antonio and Federico the following day. Antonio was still alive thus was rushed to the hospital while
Federico was already dead.

ISSUE:
1. Was the RTC correct in imposing the death penalty?
2. Was the appreciation of the aggravating circumstances proper?

People vs Castillo
PEOPLE OF THE PHILIPPINES, appellee, vs. ELIZABETH CASTILLO and EVANGELINE PADAYHAG, appellants.
[G.R. No. 132895. March 10, 2004]
Doctrine/s and/or Rulings: We affirm the trial courts judgment convicting Castillo. However, we acquit her co-accused
Padayhag.
To sustain a conviction for Kidnapping and Serious Illegal Detention under Article 267 of the Revised Penal Code,[8] the
prosecution must establish the following: (1) the offender is a private individual; (2) he kidnaps or detains another or in any
other manner deprives the victim of his liberty; (3) the act of kidnapping or detention is illegal; and (4) in the commission of
the offense any of the following circumstances is present: (a) the kidnapping or detention lasts for more than three days;
(b) it is committed by simulating public authority; (c) serious physical injuries are inflicted on the victim or threats to kill are
made; or (d) the person kidnapped or detained is a minor, female or a public officer.[9]
Appellant Castillos Liability
Castillo asserts that the victims parents did not pay her wages when she worked as a maid of the victims family.[10] She
claims that it was this injustice, her educational level and her ignorance of the law, which impelled her to take Rocky. She
faults the trial court for refusing to consider this. Castillo is mistaken. Whether or not her employer failed to pay her salary
is irrelevant. No amount of perceived injustice can serve as justification for any person to retaliate through the commission
of another crime. The trial court was therefore correct in disregarding Castillos claim that Rockys parents committed
injustice on her.
Castillos claim of injustice cannot justify in any way her demand for ransom. Ransom is money, price or consideration paid
or demanded for redemption of a captured person or persons, a payment that releases from captivity.[11] Thus, even if
she had a right to demand payment of her unpaid wages, the money she actually demanded and eventually received, is
still ransom.
Castillos reliance on her low educational level is similarly unavailing. The penalty for kidnapping for ransom is the singular
and indivisible penalty of death. This bars the application of any alternative, mitigating or aggravating circumstance
Appellant Padayhags Liability
The same cannot be said of Padayhag. Our review of the evidence on record shows that the prosecution failed to prove
Padayhags guilt beyond reasonable doubt.
We reiterate the doctrine that an appeal in a criminal case opens the entire case for review on any question including
those not raised by the parties.[25] This becomes even more imperative in cases where the penalty imposed is death.
There must be positive and conclusive evidence that Padayhag acted in concert with Castillo to commit the same criminal
act. To hold an accused guilty as a co-principal by conspiracy, there must be a sufficient and unbroken chain of events
that directly and definitely links the accused to the commission of the crime without any space for baseless suppositions
or frenzied theories to filter through.[26] Indeed, conspiracy must be proven as clearly as the commission of the crime
itself.[27]
Conspiracy is established by the presence of two factors: (1) singularity of intent; and (2) unity in execution of an unlawful
objective. The two must concur. Performance of an act that contributes to the goal of another is not enough. The act must
be motivated by the same unlawful intent. Neither joint nor simultaneous action is per se sufficient indicium of conspiracy,
unless proved to have been motivated by a common design.[28]
Padayhags act of fetching Rocky is not conclusive proof of her complicity with Castillos plan, a plan Padayhag did not
even know. Both appellants testified that Padayhag met Castillo only because Castillo told Padayhag that Padayhags
boyfriend was sick. It was precisely on the pretext that they were to visit Padayhags boyfriend that the two met. When
they met, Padayhag realized that Castillo had deceived her:
All these circumstances illustrate the absence of any hint of conspiracy. We also find that the prosecution failed to prove
Padayhags guilt beyond reasonable doubt. In People v. Gonzales[36] we held:
In the absence of conspiracy, if the inculpatory facts and circumstances are capable of two or more explanations, one of
which is consistent with the innocence of the accused and the other consistent with his guilt, then the evidence does not
fulfill the test of moral certainty and is not sufficient to support a conviction.
Penalty and Damages
Under Article 267 of the Revised Penal Code,[44] the penalty of death is imposed upon proof that the kidnapping was
committed to extort ransom from the victim or any other person. We find that the prosecution has established Castillos
guilt for this crime beyond reasonable doubt. However, Castillos pecuniary liability must be modified to conform with
jurisprudence. The award of exemplary damages must be deleted in the absence of any aggravating circumstance. Mr.
Cebrero testified that their family suffered serious anxiety at the possibility of not seeing Rocky again.[45] The pain and
anguish they experienced justifies the award of moral damages. However, we reduce the trial courts award of moral
damages to P100,000 in line with current jurisprudence.
Brief Facts: Before us on automatic review is the Decision[1] of the Regional Trial Court of Paranaque, Branch 260,
National Capital Judicial Region, in Criminal Case No. 95-86, finding appellants Elizabeth Castillo (Castillo) and
Evangeline Padayhag (Padayhag) guilty of Qualified Kidnapping and Serious Illegal Detention[2] and sentencing them to
death.
The Information[3] charging Castillo, Padayhag and Imelda Wenceslao with the crime of kidnapping, reads:
That on or about March 1, 1995, in Paraaque, Metro Manila, Philippines, and within the jurisdiction of the Honorable
Court, said accused ELIZABETH CASTILLO and EVANGELINE PADAYHAG, conspiring together, confederating, and
mutually helping one another, did then and there willfully, unlawfully and feloniously kidnap, carry away, and seriously
detain HORACIO CEBRERO IV @ Rocky, a five years old child (sic), which kidnapping or serious detention lasted for
more than three (3) days thereby depriving him of his liberty, and which was committed for the purpose of extorting
ransom from the parents of the victim, to the damage and prejudice of the victim himself and his parents.
Issues:
I. THE TRIAL COURT ERRED IN MISAPPRECIATING (SIC) THE FACTS OF THE CASE.

II. THE TRIAL COURT ERRED IN CONCLUDING THAT THERE WAS CONSPIRACY TO EXTORT RANSOM IN
THIS CASE.
III. THE TRIAL COURT ERRED IN CONSIDERING THE UNCOUNSELLED CONFESSION OF EVANGELINE
PADAYHAG.
IV. THE TRIAL COURT GRAVELY ERRED IN IMPOSING THE DEATH PENALTY IN THE CASE AT BAR.[7]

People vs Tan, G.R. No. 132324. September 28, 1999

Doctrines:
Self defense
When the accused invoke self-defense, the burden of proof is shifted to them to prove that the killing was justified and that
they incurred no criminal liability therefor. They must rely on the strength of their own evidence and not on the weakness
of that of the prosecution, for even if the latter is weak, it could not be disbelieved after their open admission of
responsibility for the killing.
In the present case, it is incumbent upon Appellant Norly Tan to prove self-defense. Thus, he must prove that there was
unlawful aggression on the part of the victim, that the means employed to prevent it were reasonable, and that there was
lack of sufficient provocation o his part. However, he failed to discharge this burden.

Crime and its Punishment


The killing was attended by treachery; hence, the crime was murder. The essence of treachery is the sudden and
unexpected attack, without the slightest provocation on the part of the person attacked. Treachery is present when the
offender commits any of the crimes against persons, employing means, methods or forms in the execution thereof, which
tend directly and especially to insure its execution, without risk arising from the defense which the offended party might
make. In the case at bar, the attack on Magdalino Olos was treacherous, because he was caught off guard and was
therefore unable to defend himself, as testified to by the prosecution witnesses and as indicated by the wounds inflicted
on him.

Culpability of Jose Tan


The prosecution was not able to establish conspiracy in the killing of the victim; thus, Appellant Jose Tan is guilty only as
an accomplice. According to the widow Ofelio Olos, she even heard Jose Tan telling and pleading with his brother to stop
his attack and stabbing of the victim. The most therefore that said accused could be liable for is merely that of an
accomplice, who, not being a principal cooperated in the execution of the offense by previous and simultaneous acts, that
in this case, by his stoning the victim Modesto Olos and hitting him on the neck. However, the accused Jose Tans act of
stoning was neither a direct participation nor indispensable to the killing of the victim. Also, as held by the Supreme Court,
when doubt exists whether an accused acted as principal or accomplice, the court should favor the lesser or milder
identity (People vs. Irenea, G.R. No. 44410, August 5, 1988).
The penalty of Appellant Jose Tan as an accomplice is one degree lower than that of the principal, which in murder cases
is reclusion temporal, in its maximum period, to death. Considering that he is entitled to the privileged mitigating
circumstance of minority, because he was only sixteen years old when the crime was committed, the trial court should
have lowered his penalty by two degrees, i.e. prision correccional maximum to prision mayor medium. Likewise, he is
entitled to the benefits of the Indeterminate Sentence Law.
Since no aggravating or mitigating circumstance was proven, the imposable penalty on Norlito Tan is reclusion perpetua.

Facts:
On the 6th day of September, 1993, in Barangay Gatbo, Municipality of Ocampo, Province of Camarines Sur, Philippines,
Norlito Tan and Jose Tan as an accomplice, with intent to kill, with treachery and evident premeditation, conspiring,
confederating together and mutually helping one another, did then and there, wilfully, unlawfully and feloniously attack,
assault, stone and stab with a deadly weapon one Magdaleno Rudy Olos alias Modesto Olos, thereby inflicting upon the
latter mortal wounds on the different parts of his body which caused his death, to the damage and prejudice of the heirs of
the offended party in such amount as may be proven in court.
On December 14, 1995, Jose Tan was arrested in Ocampo, Camarines Sur. Subsequently, Norlito Tan was arrested on
April 1, 1996. They both pleaded not guilty.

Issue:
Whether or not self defense can be used as a justifying circumstance in this case.
Whether Jose Tan can be considered as an accomplice in this case.

Case # 47 People v. Abarri


G.R. No. 90185 March 1, 1995

Art. 8 Conspiracy and proposal to commit a felony


Art. 17 Principals

Doctrine/ Ruling:
A conspiracy exists when two or more persons come to an agreement concerning the commission of a felony and decide
to commit it. Proof of the agreement need not rest on direct evidence, as the agreement itself may be inferred from the
conduct of the parties disclosing a common understanding among them with respect to the commission of the offense.

The common intent of robbing the victim and committing the acts of lasciviousness can be inferred from their behaviors.

Abarri and Andales each poked a knife at Gan's neck and forcibly brought her to the vacant lot. The other appellants
followed them and watched while Abarri divested the victim of her valuables. After robbing the victim, Abarri with the use
of a "balisong" tore the upper portion of the victim's blouse and all the other appellants participated in removing her
clothes, pawing her and biting her nipples.
The presence or absence of lewd designs is inferred from the nature of the acts themselves and the environmental
circumstances. We find that the acts of appellants in striping naked and hogtying the victim and touching her private parts
constitute lewd designs.

However, in the case of Andales, the acts of lasciviousness committed by him culminated in the raping of the victim when
he was left alone with her. Nothing in the records show that the other accused had knowledge or were aware of the rape
committed by Andales. Consequently, he alone is guilty of robbery with rape.

Likewise, we do not regard the remarks made by Cawaling to Andales as sufficient to make him a principal by
inducement or a co-conspirator. Before a remark can produce such an effect, the same must be of a nature and
uttered in such a manner as to become the determining cause of the crime, as to make the utterance a command
from a superior to a subordinate. In the case at bench, it appears that the decision of Andales to rape the victim
had been made before Cawaling uttered the remarks. Cawaling was then leaving the place with Abarri, Estrada
and Pajalago while Andales purposely stayed behind with the victim. There is not even a showing that Cawaling
had any moral influence over Andales.
Facts:
On October 14, 1988, at around 7:30 P.M., while Gregoria Gan was walking along 4th Avenue, Kalookan City on her way
home, Ernesto Abarri and Ronnie Andales stopped her and each poked a knife at her neck. Abarri then grabbed Gan's
bag and warned: "Kung gusto mong mabuhay, huwag kang sisigaw." Gan was dragged by the two and brought inside a
fenced, vacant lot strewn with garbage and covered with tall grass. Clemente Cawaling, Conrado Estrada and Joselito
Pajalago were former employees of Gan.
Once inside the vacant lot, Abarri, with the use of a "balisong, " tore the upper portion of Gan's blouse. The other accused
then started tearing the rest of the blouse and pulling down her pants. The torn blouse was used to tie her mouth, hands
and feet. When she was completely naked, the accused started touching her private parts.
Abarri opened Gan's handbag and took a bunch of keys, which included the key for her store at Carmen Planas Street in
Binondo, Manila. He also got her watch valued P2,000.00, necklace valued at P5,000.00 and wallet containing P250.00.

After robbing Gan, appellants left except Andales. Before leaving, Cawaling told Andales: "Nognog, (referring to
Andales) bahala ka na, sampung taon na rin na hindi nakakatikim 'yan, makatas pa 'yan."

Andales then dragged Gan to a dark spot and after loosening the tie on her legs, raped her twice. After satisfying his lust,
Andales left.

Gan waited for about 20 minutes before she started to roll over to the middle of the lot. In the process, the tie on her
mouth loosened and she was able to shout for help. Responding to her cries, neighbors came and untied her hands.

Meanwhile, at around 8:30 P.M. of the same day, Barangay Captain Anita Alejo was informed by a resident that somebody
was opening the store of Gan. Repairing at the place, Alejo saw Abarri and Estrada. She noticed that the door of the store
had been partly opened. When she asked the two what they were doing there, Abarri answered that Gan instructed them
to get the latter's pants. Alejo brought them to the barangay hall for investigation. Upon further questioning, Abarri
admitted to forcibly bringing Gan to a vacant lot and binding her arms and legs. Alejo turned over the two to the custody of
the police detachment in Binondo.

The police brought Abarri and Andales to the crime scene. However, Gan was no longer there when they arrived. The
police proceeded to Gan's house where the latter positively identified the two as among those persons who robbed her.

Issue:
Are the accused guilty of robbery with rape?

Case No. 48
Enrile vs. Salazar
186 SCRA 217
G.R. No. 92163 – 64
June 5, 1990

Intro:
Article 48 of our Penal Code cannot be applied in the case at bar. If murder were not complexed with rebellion, and the
two crimes were punished separately (assuming that this could be done), the following penalties would be imposable
upon the movant, namely: (1) for the crime of rebellion, a fine not exceeding P20,000 and prision mayor, in the
corresponding period, depending upon the modifying circumstances present, but never exceeding 12 years of prision
mayor, and (2) for the crime of murder, reclusion temporal in its maximum period to death, depending upon the modifying
circumstances present. in other words, in the absence of aggravating circumstances, the extreme penalty could not be
imposed upon him. However, under Article 48 said penalty would have to be meted out to him, even in the absence of a
single aggravating circumstance. Thus, said provision, if construed in conformity with the theory of the prosecution, would
be unfavorable to the movant.

Facts:
A warrant was issued charging Senator Enrile, the spouses Rebecco and Erlinda Panlilio, and Gregorio Honasan with the
crime of rebellion with murder and multiple frustrated murder allegedly committed during the period of the failed coup
attempt from November 29 to December 10, 1990. The petitioner's counsel contended that the petitioner is charged with a
crime that does not exist in the statute books, therefore, the same must be dismissed as it is just a mere flight of rhetoric.

Issue:
Whether or not under Article 48 of the Revised Penal Code rebellion may properly be complexed with a common offense,
murder.

#49 Santiago v Garchitorena

ART 10 RPC - Supplementing Special Laws

CONTINUING CRIME - For delito continuado to exist there should be a plurality of acts performed during a period of time;
unity of penal provision violated; and unity of criminal intent or purpose, which means that two or more violations of the
same penal provisions are united in one and same instant or resolution leading to the perpetration of the same criminal
purpose or aim

a delito continuado consists of several crimes but in reality there is only one crime in the mind of the perpetrator.
FACTS:

Petitioner was charged with performing a single criminal act — that of her approving the application for legalization of 32
aliens not qualified under the law to enjoy such privilege under EO 324, causing undue injury to one offended party, the
Government, done on a single day, i.e., on or about October 17, 1988.

According to petitioner, unless she was furnished with the names and identities of the aliens, she could not properly plead
and prepare for trial.

The information was then amended reproducing verbatim the allegation of the original information, except that instead of
the word "aliens" in the original information each amended information states the name of the individual whose stay was
legalized.

At the hearing of the motion for a bill of particulars, the public prosecutors manifested that they would file only one
amended information embodying the legalization of stay of the 32 aliens. As stated in the Order dated November 12, 1992
of the Sandiganbayan (First Division):

On the matter of the Bill of Particulars, the prosecution has conceded categorically that the accusation against Miriam
Defensor Santiago consists of one violation of the law represented by the approval of the applications of 32 foreign
nationals for availment (sic) of the Alien Legalization Program. In this respect, and responding directly to the concerns of
the accused through counsel, the prosecution is categorical that there will not be 32 accusations but only one . . . (

ISSUE:
Is the consolidation of cases proper for violation of EO 324? / Was there only one offense committed?

RULING:
YES. Under Article 10 of the Revised Penal Code, the Code shall be supplementary to special laws, unless the latter
provide the contrary. Hence, legal principles developed from the Penal Code may be applied in a supplementary capacity
to crimes punished under special laws.

Applying the concept of delito continuado

The 32 Amended Informations aver that the offenses were committed on the same period of time, i.e., on or about
October 17, 1988. The strong probability even exists that the approval of the application or the legalization of the stay of
the 32 aliens was done by a single stroke of the pen. Likewise, the public prosecutors manifested at the hearing the
motion for a bill of particulars that the Government suffered a single harm or injury.

Criminal Case No. 16698 is MODIFIED in the sense that the Office of the Special Prosecutor of the Office of the
Ombudsman is directed to consolidate the 32 Amended Informations into one information charging only one offense under
the original case number,
Case # 52
Cristobal v. Labrador
G.R. No. L-47941 December 7, 1940

ABSOLUTE PARDON
DOCTRINE: Absolute pardon not only blots out the crime committed but removes all disabilities resulting from the
conviction.
FACTS: The CFI of Rizal found respondent Santos guilty of estafa, however, he continued to be a registered elector in the
city of Malabon, Rizal and was seated as the municipal president from 1934 to 1937. On 1938, Commonwealth Act No
357 or the Election Code was approved which disqualifies the respondent from voting for having been “declared by final
judgment guilty of any crime against property.” The respondent applied to the President for an absolute pardon and
granted the petition restoring the respondent to his full civil and political rights, except that with respect to the right to hold
public office or employment, he will be eligible for appointment only to positions which are clerical or manual in nature and
involving no money or property responsibility.

On 1940, Cristobal filed a petition for the exclusion of the name of Santos from the list of voters in precinct no. 11 of
Malabon, Rizal on the ground that the latter is disqualified. After hearing, the court denied the petition for exclusion and
declared that the pardon extended in favor of respondent has had the effect of excluding him from the disqualification
created by the Election Code.

ISSUE: Whether or not the pardon not only blots out the crime committed but removes all disabilities resulting from the
conviction?

54. RICARDO PARULAN, petitioner, vs. DIRECTOR OF PRISONS, respondent. G.R. No. L-28519 February 17, 1968
Ruling: There are crimes which are called transitory or continuing offenses because some acts material and essential to
the crime occur in one province and some in another, in which case, the rule is settled that the court of either province
where any of the essential ingredients of the crime took place has — jurisdiction to try the case. There are, also, crimes
which although all the elements thereof for its consummation may have occurred in a single place, yet by reason of the
very nature of the offense committed, the violation of the law is deemed to be continuing. The crime of evasion of service
of sentence is an example of the latter kind of crime - when the prisoner in his attempt to evade the service of the
sentence imposed upon him by the courts and thus defeat the purpose of the law, moves from one place to another; for, in
this case, the act of the escaped prisoner is a continuous or series of acts, set on foot by a single impulse and operated
by an unintermittent force, however long it may be. It may not be validly said that after the convict shall have escaped
from the place of his confinement the crime is fully consummated, for, as long as he continues to evade the service of his
sentence, he is deemed to continue committing the crime, and may be arrested without warrant, at any place where he
may be found. Rule 113 of the Revised Rules of Court may be invoked in support of this conclusion, for, under section 6[c]
thereof, one of the instances when a person may be arrested without warrant is where he has escaped from confinement.
Undoubtedly, this right of arrest without a warrant is founded on the principle that at the time of the arrest, the escapee is
in the continuous act of committing a crime — evading the service of his sentence.
ISSUE: Whether or not the Court of First Instance of Manila has jurisdiction to hear and decide the case against Parulan?
FACTS: Petitioner Parulan was confined in the state penitentiary at Muntinlupa, Rizal, serving a sentence of life
imprisonment which, however, was commuted to 20 years by the President. He was then transferred to Fort Bonifacio in
Makati. In October 1964, while still serving his sentence, he escaped. He was eventually recaptured in Manila.
Consequently, he was prosecuted for the crime of evasion of service of sentence and was found guilty. As a recourse, he
filed a petition for the issuance of writ of habeas corpus directed against respondent Director of Prisons.

PEOPLE OF THE PHILIPPINES vs. RENATO TALUSAN y PANGANIBAN


G.R. No. 179187 July 14, 2009
CARPIO MORALES, J.:

Doctrine(s): Special complex crime. Where the law provides a single penalty for two or more component offenses, the
resulting crime is called a special complex crime. In a special complex crime, the prosecution must necessarily prove
each of the component offenses with the same precision that would be necessary if they were made the subject of
separate complaints.
Mitigating Circumstance of voluntary plea of guilty in single and indivisible penalty. The presence of mitigating
circumstance of voluntary plea of guilty has no effect or disregarded incase the penalty imposable is single and indivisible.
Automatic Review in case the penalty imposed is death, reclusion perpetua or life imprisonment. When the penalty
of death, reclusion perpetua or life imprisonment is imposed, the case is automatically forwarded to the Supreme Court on
automatic review. However, such automatic review to the Supreme Court does not bar the referring of the case to the
Court of Appeals for intermediate disposition.
Searching inquiry incase of plea of guilty in a crime having a penalty of death, reclusion perpetua or life
imprisonment. There is thus no hard and fast rule as to how a judge may conduct a " searching inquiry." As long as the
voluntary intent of the accused and his full comprehension of the consequences of his plea are ascertained.

Facts: In January 14, 2004, while AAA was on her way to school, appellant kidnapped AAA by deceiving the latter that
they would would go to Jollibee, but in fact appellant brought AAA to a house in Imus, Cavite. AAA was thereafter under
appellant’s control and custody for eight days during which he abused her by inserting his finger inside her vagina on a
daily basis before breakfast, despite her resistance.
AAA having failed to return home, her stepfather BBB went to search for the former. A neighbor then informed him that he
saw appellant with AAA while the latter was on her way to school. At dawn of the following day, January 23, 2004,
appellant, who was with AAA, was apprehended.
An information for kidnapping with rape was filed against appellant by on the basis of the report submitted by the medico
legal.
Upon arraignment, appellant, with the assistance of his counsel de oficio, entered a plea of guilty. The lower court
thereupon conducted a searching inquiry into the voluntariness of appellant’s plea, and despite repeated questions and
just as repeated answers showing that appellant understood his plea and its consequences, the trial court still ordered the
prosecution to, as it did, present evidence.
Issue: Whether or not a searching inquiry is required in case of plea of guilty where the penalty imposed is death,
reclusion perpetua or life imprisonment?

G.R. No. 198554 July 30, 2012


MAJOR GENERAL CARLOS F. GARCIA, AFP (RET.), Petitioner,
vs.
THE EXECUTIVE SECRETARY,

Doctrines:
1. A court-martial case is a criminal case and the General Court Martial is a “court” akin to any other courts. The
General Court Martial is a court within the strictest sense of the word and acts as a criminal court. On that
premise, certain provisions of the Revised Penal Code, insofar as those that are not provided in the Articles of
War and the Manual for Courts-Martial, can be supplementary. Under Article 10 of the Revised Penal Code: Art.
10. Offenses not subject to the provisions of this Code.—Offenses which are or in the future may be punishable
under special laws are not subject to the provisions of this Code. This Code shall be supplementary to such laws,
unless the latter should specially provide the contrary.

2. A special law is defined as a penal law which punishes acts not defined and penalized by the Revised Penal
Code. In the present case, petitioner was charged with and convicted of Conduct Unbecoming an Officer and
Gentleman (96th Article of War) and Violation of the 97th Article of War, or Conduct Prejudicial to Good Order and
Military Discipline, both of which are not defined and penalized under the Revised Penal Code. The corresponding
penalty imposed by the General Court Martial, which is two (2) years of confinement at hard labor is penal in
nature. Therefore, absent any provision as to the application of a criminal concept in the implementation and
execution of the General Court Martial's decision, the provisions of the Revised Penal Code, specifically Article 29
should be applied.
3. Article 29 of the Revised Penal Code in the Articles of War is in accordance with the Equal Protection Clause of
the 1987 Constitution. According to a long line of decisions, equal protection simply requires that all persons or
things similarly situated should be treated alike, both as to rights conferred and responsibilities imposed. It
requires public bodies and institutions to treat similarly situated individuals in a similar manner. The purpose of the
equal protection clause is to secure every person within a state’s jurisdiction against intentional and arbitrary
discrimination, whether occasioned by the express terms of a statute or by its improper execution through the
state’s duly-constituted authorities

Facts:
Major General Carlos Garcia of the AFP was charged in a Court Martial for failure to disclose all his existing assets in his
Sworn Statement of Assets and Liabilities and Net Worth for the years 2002 and 2003. After six (6) years and two (2)
months of preventive confinement, Garcia was found guilty of the charged offenses and was dishonorably discharged
from the service, forfeited all pay and allowances due and confined at hard labor at such place the reviewing authority
may direct for a period of two (2) years.
Issue: WON the period of preventive confinement of Garcia shall be credited to the sentence imposed by the court
martial.
Ruling: Yes. Applying the provisions of Article 29 of the Revised Penal Code, the time within which the petitioner was
under preventive confinement should be credited to the sentence confirmed by the Office of the President. The period of
confinement of six years shall be credited in his favor and deducted from the two (2) years to which the accused was
sentenced.

FERDINAND A. CRUZ, Petitioner, v. THE PEOPLE OF THE PHILIPPINES, Respondnet.


G.R. No. 176504, Spetember 03, 2008

ARTICLE
 Article 310 of the Revised Penal Code, theft is qualified when it is, among others, committed with grave abuse of
confidence, to wit:

ART. 310. Qualified theft. - The crime of theft shall be punished by the penalties next higher by two
degrees than those respectively specified in the next preceding article, if committed x x x with grave
abuse of confidence x x x.

 The settled rule is that when an accused pleads to the charge, he is deemed to have waived the right to
preliminary investigation and the right to question any irregularity that surrounds it.

 Under Article 310 of the Revised Penal Code, the penalty for Qualified Theft is two degrees higher than that
specified in Article 309.

FACTS:
Ferdinand Cruz was a Marketing Manager of Porta-Phone Rentals, Inc., a corporation engaged in the lease of cellular
phones. He went to his office, obtained a pad of official receipts from the collection officer’s table. He delivered the
communication equipment and received the PhP15,000.00 payment and issued a receipt even though he not authorized
to receive cash payments and issued receipts. He then failed to deliver the cash to his office. When he was confronted,
Ferdinand admitted that he deposited the amount to his personal bank account. He was instructed to remit the amount
which he failed to do claiming that the company has paid his reimbursements.. The company sent a demand letter which
he answered stating that he already remitted the amount to the accounting supervisor which the latter denied. He refused
to turn over the amount despite demands made by the officers of the company which prompted the company to file the a
case of Estafa/Falsification of Documents. According to the Information filed before the RTC of Makati, Ferdinand Cruz as
a Marketing Manager of Porta-Phone Rentals, Inc. had access to the funds of the corporation and with grave abuse of
confidence, steal and carry away the amount of PhP15,000.00. He entered a plea of not guilty during arraignment, and
the witnesses for the prosecution testified that he received the said amount from Hemisphere-Leo Burnett and issued a
receipt but failed to turn over the amount to Porta-Phone Rentals. The prosecution changed the case to Qualified Theft.
Ferdinand Cruz in his defense alleged that he issued an acknowledgment receipt to Hemisphere and remitted the amount
to the accounting supervisor. He was asked by the supervisor to sign the official receipt because he was the one who
closed the deal.
The RTC found Ferdinand guilty beyond reasonable doubt for the crime of QUALIFIED THEFT. He filed a Motion for New
Trial on the grounds of (1) absence of a preliminary investigation for the crime of qualified theft and (2) newly discovered
evidence – the testimony of a former employee of Hemisphere testifying on Ferdinand’s signing of an acknowledgment
receipt. The RTC revived and reinstated the conviction of Ferdinand. Upon appeal, the Court of Appeals affirmed the RTC
Decision.

ISSUE:
1. Whether Ferdinand was denied of due process when he was indicted for qualified theft even as he was initially
investigated for estafa/falsification of private documents.
2. Whether Ferdinand’s guilt was not established beyond reasonable doubt.
3. Whether the Indeterminate Sentence Law should be applied.

HELD:
1. Ferdinand was not denied of due process. The settled rule is that when an accused pleads to the charge, he is
deemed to have waived the right to preliminary investigation and the right to question any irregularity that
surrounds it. In the instant case, Ferdinand did not present evidence that arraignment was forced upon him. On
the contrary, he voluntarily pleaded to the charge and actively participated in the trial of the case

It is not correct for Ferdinand to claim that preliminary investigation on the charge of qualified theft was not
accorded him. The truth is, Ferdinand was able to answer the initial charge of estafa/falsification of private
documents through his counter-affidavits. Based on the same complaint affidavit and the same sets of evidence
presented by the complainant, the prosecutor deemed it proper to charge Ferdinand with qualified theft. Since the
same allegations and evidence were proffered by the complainant in the qualified theft, there is no need for
Ferdinand to be given the opportunity to submit counter-affidavits anew, as he had already answered said
allegations when he submitted counter-affidavits for the original indictment of estafa/falsification of private
documents.

2. The elements of the crime of theft are the following: (1) there was a taking of personal property; (2) the property
belongs to another; (3) the taking was without the consent of the owner; (4) the taking was done with intent to
gain; and (5) the taking was accomplished without violence or intimidation against the person or force upon
things.12 Under Article 310 of the Revised Penal Code, theft is qualified when it is, among others, committed with
grave abuse of confidence, to wit:

ART. 310. Qualified theft. - The crime of theft shall be punished by the penalties next higher by two degrees than
those respectively specified in the next preceding article, if committed x x x with grave abuse of confidence x x x.

The prosecution established, beyond the shadow of doubt that Ferdinand took and kept the fifteen thousand
peso-collection from the company’s client.

3. The RTC imposed on petitioner the indeterminate penalty of 10 Years and 1 Day of prision mayor as minimum to
14 Years, 8 Months and 1 Day of reclusion temporal, as maximum.

Under Article 310 of the Revised Penal Code, the penalty for Qualified Theft is two degrees higher than that
specified in Article 309. Paragraph 1 of Article 309 provides that if the value of the thing stolen is more than
P12,000.00 but does not exceed P22,000.00, the penalty shall be prision mayor in its minimum and medium
periods. In this case, the amount stolen was P15,000.00. Two degrees higher than prision mayor minimum and
medium is reclusion temporal in its medium and maximum periods. Applying the Indeterminate Sentence Law, the
minimum shall be prision mayor in its maximum period to reclusion temporal in its minimum period or within the
range of 10 years and 1 day to 14 years and 8 months. There being neither aggravating nor mitigating
circumstance in the commission of the offense, the maximum period of the indeterminate sentence shall be within
the range of 16 years, 5 months and 11 days to 18 years, 2 months and 20 days. The minimum penalty imposed
by the RTC is correct. However, the maximum period imposed by RTC should be increased to 16 years, 5 months
and 11 days.

PEOPLE OF THE PHILIPPINES v. ARNULFO ASTORGA


G.R. No. 110097, 22 December 1997

Introduction (Provisions involved, principles, jurisprudence):

Actual detention or "locking up" is the primary element of kidnapping. If the evidence does not adequately prove this
element, the accused cannot be held liable for kidnapping. In the present case, the prosecution merely proved that
appellant forcibly dragged the victim toward a place only he knew. There being no actual detention or confinement, the
appellant may be convicted only of grave coercion.

No Motive to "Kidnap"

Motive is not an element of the crime. Furthermore, motive becomes material only when the evidence is circumstantial or
inconclusive, and there is some doubt on whether a crime has been committed or whether the accused has committed it.
Indeed, motive is totally irrelevant when ample direct evidence sustains the culpability of the accused beyond reasonable
doubt. In this case, the identity of appellant is not in question. He himself admitted having taken Yvonne to Maco Central
Elementary School.

Kidnapping or Coercion?
The prosecution failed to prove one essential element of kidnapping — the fact of detention or the deprivation of liberty.
Under Article 267 of the Revised Penal Code, the elements of kidnapping are as follows:

1. That the offender is a private individual.


2. That he kidnaps or detains another, or in any other manner deprives the latter of his liberty.
3. That the act of detention or kidnapping must be illegal.
4. That in the commission of the offense, any of the following circumstances is present:
1. That the kidnapping or detention lasts for more than five (5) days; or
2. That it committed simulating public authority; or
3. That any serious physical injuries are inflicted upon the person kidnapped or detained or threats to
kill him are made; or
4. That the person kidnapped or detained is a minor, female, or a public officer.

The Spanish version of Article 267 of the Revised Penal Code uses the terms "lockup" ( encerrar) rather than "kidnap"
(secuestrar or raptar). Lockup is included in the broader term of "detention," which refers not only to the placing of a
person in an enclosure which he cannot leave, but also to any other deprivation of liberty which does not necessarily
involve locking up. Likewise, the Revised Penal Code was originally approved and enacted in Spanish. Consequently, the
Spanish text is controlling in cases of conflict with the English version, as provided in Section 15 of the Revised
Administrative Code.

A review of the events as narrated by the prosecution witnesses ineluctably shows the absence of "locking up." It is clear
that the appellant and the victim were constantly on the move. There is no actual confinement or restraint of the victim,
which is the primary element of kidnapping. Appellant's apparent intention was to take Yvonne against her will towards the
direction of Tagum. The evidence does not show that appellant wanted to detain Yvonne; much less, that he actually
detained her. Appellant's forcible dragging of Yvonne to a place only he knew cannot be said to be an actual confinement
or restriction on the person of Yvonne. There was no "lockup." Accordingly, appellant cannot be convicted of kidnapping
under Article 267 of the Revised Penal Code.

Rather, the felony committed in this case is grave coercion under Article 286 of the same code. Grave coercion
or coaccion grave has three elements: (a) that any person is prevented by another from doing something not prohibited by
law, or compelled to do something against his or her will, be it right or wrong; (b) that the prevention or compulsion is
effected by violence, either by material force or such a display of it as would produce intimidation and, consequently,
control over the will of the offended party; and (c) that the person who restrains the will and liberty of another has no right
to do so or, in other words, that the restraint is not made under authority of a law or in the exercise of any lawful
right. When appellant forcibly dragged and slapped Yvonne, he took away her right to go home to Binuangan. Appellant
presented no justification for preventing Yvonne from going home.

Brief facts:

While there was a brown out, accused or "Boy" Astorga told Yvonne Traya, who was only eight (8) years old at that time
and who stays with her grandparents and so with her parents at Sitio Binuangan, Maco, to go with him to buy candy and
immediately grabbed and held Yvonne’s hand as the latter did not answer, placed his hand on her shoulder, covered her
mouth, and went and walked inside the compound of Maco Elementary School, and later, there being no person around
the gate, accused brought her out to the highway and walked towards the direction of Tagum which is the opposite
direction towards her grandparent's house at Binuangan, hence Yvonne cried and protested that she must go home but
the accused did not heed her plea.

While the accused and Yvonne were walking in the situation as described, somewhere near the Luponlupon bridge, they
met some group of youngsters-men and the said group was suspicious about the man who was bringing a child, thus the
said group decided to follow them. Accused hurriedly walked fast with Yvonne, and to prevent from being overtaken, he
carried the victim and ran but they were chased and were overtaken.

Edwin Fabila declared that Jonathan, one of his companions with others in chasing, asked the accused where they were
bound and he answered towards Binuangan but the group noticed something suspicious because their destination was
already towards Tagum which is an opposite direction to Binuangan.

Jonathan, one of those who chased, knew Yvonne’s family and he got from the accused Yvonne who showed some
resistance, and the group brought the accused and Yvonne to Yvonne’s home at Binuangan.

Brief issues:
Whether the accused’s motive to kidnap Yvonne Traya is relevant to convict him for the crime of kidnapping. No, motive
is not relevant.

Whether Yvonne Traya was not detained, locked-up or deprived of her liberty so as not to convict the accused for the
crime of kidnapping but only for grave coercion. Yes!

ORTEGA V. PEOPLE (CRIMINAL)

ARTICLE 12 - EXEMPTING CIRCUMSTANCES

Section 6 of RA 9344 clearly and explicitly provides:

Section 6. Minimum Age of Criminal Responsibility - A child 15 years of age or under at the time of the commission of the
offense shall be exempt from criminal liability. However, the child shall be subjected to an intervention program pursuant to
Section 20 of this Act.

A child above 15 years but below 18 years of age shall likewise be exempt from criminal liability and be subjected to an
intervention program, unless he/she acted with discernment, in which case, such child shall be subjected to the
appropriate proceedings in accordance with this Act

ORTEGA vs PEOPLEGR No. 151085August 20, 2008

FACTS:
The petitioner, Joemar Ortega, who was then 14 years old, was charged with the crime of rape
for allegedly raping AAA, who was about 8 years old. That the rape happened in 3 occasions. The RTC ruled that the petitioner is
guilty beyond reasonable doubt in the crime of rape and is sentenced to reclusion temporal. The CA affirmed the ruling of the trial
court. During the pendency of the case in the SC, RA 9344 Juvenile Justice and Welfare Act was enacted that establishes a comprehensive
system to manage children in conflict with the law. At the case at bar, because the petitioner was a minor under 15 years of age at the
commission of the crime, he can be relieved from criminal liability.

ISSUE:
Whether or not the petitioner can avail exempting circumstance provided by the newly enacted law on minors in conflict with law.
HELD
: Yes
RATIO
: The petitioner can avail the exempting circumstance that will relieve him from criminal liability because the law enacted was favorable to the
accused, and is therefore retroactive in application. Juvenile Justice and Welfare Act provides that a child under 15 years of age in the
commission of the offense shall be exempt from criminal liability, but is subject to an intervention program. Exemption from criminal liability,
however, does not include exemption from civil liability. Section 64 of the newly enacted law also provides that cases of children under 15
years of age at the commission of the crime, shall immediately be dismissed and the child shall be referred to the appropriate
local social welfare and development officer. The Court therefore held that the case against Joemar Ortega is hereby
DISMISSED. Petitioner is hereby referred to the local social welfare and development officer of the locality for the appropriate
intervention program

Case No. 65

PEOPLE OF THE PHILIPPINES vs. IRVIN TADULAN y EPAN


G.R. No. 117407 | April 15, 1997
PADILLA, J.

DOCTRINE / RULING:
Alibi; It has been held time and again that for alibi to prosper as a defense the accused must show that he was so far
away that he could not have been physically present at the place of the crime, or its immediate vicinity at the time of its
commission (People vs. Tasurra, 192 SCRA 266). In this case, however, it is not so situated, for according to him he was
at the plant of the Republic Asahi Glass Corporation in Barangay Pinagbuhatan, Pasig, —which is but a few kilometers
from Barangay Sumilang of the same municipality where the crime was committed.

Rape; Accused-appellant tries to discredit the victim’s testimony by questioning her behavior after she was allegedly
raped. The court ruled that it is not proper to judge the actions of children who have undergone traumatic experience by
the norms of behavior expected under the circumstances from mature people. The range of emotion shown by rape
victims is yet to be captured even by the calculus. It is thus unrealistic to expect uniform reactions from rape victims.It
should be borne in mind, in this connection, that the victim was only a naive nine (9) year old child when the crime was
committed on her. She considered the accused as a friend, almost like a relative, as in fact she called him “Tito Loloy.”

As correctly observed by the Solicitor General: “(A)s regards the acts imputed to Estela, the delay of seven (7) days from
the date of her knowledge of the rape incident on 4 April 1992 in reporting to the authorities the rape of her daughter is
excusable. At that time, she was not yet certain of the steps she would take considering the delicate nature of the problem
they were facing” (citing People v. Danguilan, 218 SCRA 98; People v. Joaquin, Jr., 225 SCRA 179). Besides, we have
ruled that a delay in prosecuting the rape is not indicative of fabricated charges.

Pardon; It is clear to the mind of this Court that the complainant has not expressly pardoned the said accused. Besides,
there are authorities holding that pardon must be granted not only by the parents of an offended minor but also by the
minor herself in order to be effective as an express pardon under Art. 344 of the Revised Penal Code. Thus, in the case of
People vs. Lacson, Jr., (C.A.) 55 O.G. 9460, we find the following words: ‘Neither must we be understood as supporting
the view that the parents alone can extend a valid pardon. Far from it, for we, too are of the belief that the pardon by the
parents, standing alone, is inefficacious.’ It was also held in another case, that ‘The express pardon of a person guilty of
attempted abduction of a minor, granted by the latter’s parents, is not sufficient to remove criminal responsibility, but must
be accompanied by the express pardon of the girl herself.’ (U.S. vs. Luna, 1 Phil. 360).

FACTS:
Complainant Estela Santos owns a house at No. 6 Dr. Garcia St., in Barangay Sumilang, Pasig, she resides with her
common-law husband and their minor daughter, Maristel Cruz. Behind the said house, complainant also owns a three-
door apartment building, one unit of which was rented and occupied by accused Irvin Tadulan, his wife Adefa Tadulan and
their three children name Dianne, Angie and Bochoy who were aged 10, 9 and 5, respectively. In 1992 complainant’s
daughter, Maristel Cruz was about nine (9) year old and was in grade school. She often played with the accused’s children
in the vicinity of their house and the apartment building.

That on or about the 2nd day of April, 1992 in the Municipality of Pasig, Metro Manila, Philippines and within the
jurisdiction of this Honorable Court, armed with a knife, with lewd design and by means of force, threats and intimidation,
did then and there willfully, unlawfully and feloniously have sexual intercourse with one Maristel Cruz, a minor, nine (9)
years old, without her consent and against her will.

Estela Santos immediately informed the wife of Irvin Tadulan that her husband has raped her daughter. She further
informed Adefa Tadulan that she would not take action against the latter’s husband if they would vacate the apartment unit
right away. Adefa Tadulan later on met with Estela Santos and told her that she had driven away Irvin Tadulan, but
requested that she and her children be allowed to stay until Saturday, April 11, 1992. Estela Santos thereafter noted,
however, that Irvin Tadulan was still coming home to the apartment unit every night despite the promise of his wife that
she herself would call the police should he ever come back to the place.

ISSUE:
Whether or not the court erred in disregarding the defense of pardon and alibi of the accused?
PANAGUITON, JR. v. DOJ
Facts:
In 1992, Rodrigo Cawili borrowed various sums of money from petitioner Panaguiton, Jr. On January 8, 1993, Cawili and
Ramon C. Tongson (Tongson) jointly issued in favor of Panaguiton, Jr. three (3) checks in payment of the said loans. Upon
presentment for payment on March 18, 1993, the checks were dishonored.
On August 24, 1995, Panaguiton, Jr. filed a complaint against Cawili and Tongson for violating Batas Pambansa (B.P.)
Bilang 22 before the Quezon City Prosecutor's Office. However, on March 15, 1999, Assistant City Prosecutor Sampaga
held that the case had already prescribed pursuant to Act No. 3326, as amended, which provides that violations penalized
by B.P. Blg. 22 shall prescribe after four (4) years starting on the date the checks were dishonored (March 18, 1993)
Panaguiton, Jr. appealed to the DOJ but it dismissed the case on the same grounds of prescription.
The DOJ explained that Act No. 3326 applies to violations of special acts that do not provide for a prescriptive period for
the offenses thereunder. Since B.P. Blg. 22, as a special act, does not provide for the prescription of the offense it defines
and punishes, Act No. 3326 applies to it, and not Art. 90 of the Revised Penal Code which governs the prescription of
offenses penalized thereunder. The DOJ also cited the case of Zaldivia v. Reyes, Jr., wherein the SC ruled that the
proceedings referred to in Act No. 3326, as amended, are judicial proceedings, and not the one before the prosecutor's
office (administrative proceedings). Therefore, the filing of a complaint with the Office of the City Prosecutor of Quezon
City does not interrupt the running of the prescriptive period for violation of B.P. Blg. 22. Hence, this Petition for Review.

Issues:
Whether or not the offense of violation of B.P. Blg. 22 has already prescribed per Act No. 3326

Ruling:
NO.
As correctly stated by the DOJ, Act No. 3326 is the law applicable to offenses under special laws which do not provide for
their own prescriptive periods. However, as ruled in the cases of Ingco v. Sandiganbayan, Sanrio Company Limited v.
Lim, and Securities and Exchange Commission v. Interport Resources Corporation, et al, the Court ruled that the
prescriptive period is interrupted by the institution of proceedings for preliminary investigation against the accused.
Although Panaguiton, Jr. went through the proper channels, within the prescribed periods, an aggregate period of nine (9)
years had elapsed from the time he filed his complaint-affidavit up to the time the DOJ issued the assailed resolution.
Aggrieved parties, especially those who do not sleep on their rights and actively pursue their causes, should not be
allowed to suffer unnecessarily further simply because of circumstances beyond their control. Notwithstanding whether or
not the proceeding is a judicial proceeding or an administrative proceeding, Panaguiton, Jr.’s filing of the complaint
interrupted the running of the prescriptive period.

PRESIDENTIAL COMMISSION ON GOOD GOVERNMENT (PCGG) vs. HON. ANIANO A. DESIERTO, ET AL.
Prescriptive period for Anti-Graft and Corrupt Practices Act (RA No. 3019, as amended)
The applicable law in the computation of the prescriptive period is Section 2 of Act No. 3326, as amended (Act
Establishing Prescriptive Periods for Violations of Special Laws and Municipal Ordinances), which provides that
prescription shall begin to run from the day of the commission of the violation of the law, and if the same not be known at
the time, from the discovery thereof and the institution of judicial proceedings for its investigation and punishment.
overwhelm
The prescription shall be interrupted when proceedings are instituted against the guilty person, and shall begin to run
again if the proceedings are dismissed for reasons not constituting jeopardy.
In cases involving violations of R.A. No. 3019 committed prior to the February 1986 Edsa Revolution that ousted
President Ferdinand E. Marcos, we ruled that the government as the aggrieved party could not have known of the
violations at the time the questioned transactions were made (PCGG vs. Desierto, G.R. No. 140232, January 19, 2001,
349 SCRA 767; Domingo v. Sandiganbayan, supra, Note 14; Presidential Ad Hoc Fact Finding Committee on Behest
Loans v. Desierto, supra, Note 16). Moreover, no person would have dared to question the legality of those transactions.
Thus, the counting of the prescriptive period commenced from the date of discovery of the offense in 1992 after an
exhaustive investigation by the Presidential Ad Hoc Committee on Behest Loans.-Presidential Ad Hoc Fact-Finding
Committee on Behest Loans v. Desierto (2001)

RULING: Respondent Ombudsman committed grave abuse of discretion in dismissing the subject complaint on the
ground of prescription.
Respondents members of the PNB Board of Directors and Officers of NOCOSII are charged with violation of R.A. No.
3019, a special law. Amending said law, Section 4, Batas Pambansa Blg. 195, increased the prescriptive period from ten
to fifteen years.
The issue of prescription has long been laid to rest in the aforementioned Presidential Ad Hoc Fact-Finding Committee on
Behest Loans v. Desierto, where the Court held that it was well-nigh impossible for the State, the aggrieved party, to have
known the violations of R.A. No. 3019 at the time the questioned transactions were made because, as alleged, the public
officials concerned connived or conspired with the "beneficiaries of the loans. Thus, we agree with the COMMITTEE that
the prescriptive period for the offenses with which respondents in OMB-0-96-0968 were charged should be computed
from the discovery of the commission thereof and not from the day of such commission.
The assertion by the Ombudsman that the phrase 'if the same not be known' in Section 2 of Act No. 3326 does not mean
'lack of knowledge' but that the crime 'is not reasonably knowable' is unacceptable, as it provides an interpretation that
defeats or negates the intent of the law, which is written in a clear and unambiguous language and thus provides no room
for interpretation but only application.
As to when the period of prescription was interrupted, the second paragraph of Section 2, Act No. 3326, as amended,
provides that prescription is interrupted 'when proceedings are instituted against the guilty person.
Records show that the act complained of was discovered in 1992. The complaint was filed with the Office of the
Ombudsman on April 5, 1995, or within three (3) years from the time of discovery. Thus, the filing of the complaint was
well within the prescriptive period of 15 years.
FACTS: On October 8, 1992, President Fidel V. Ramos issued Administrative Order No. 13 creating the Presidential Ad
Hoc Fact-Finding Committee on Behest Loans (Committee) which was tasked to inventory all behest loans, determine the
parties involved and recommend whatever appropriate actions to be pursued thereby and Memorandum Order No. 61
expanded the functions of the Committee to include the inventory and review of all non-performing loans, whether behest
or non-behest.
Among the accounts referred to the Committee's Technical Working Group (TWG) were the loan transactions between
Northern Cotabato Sugar Industries, Inc. (NOCOSII) and Philippine National Bank (PNB) and the Committee classified the
loans obtained by NOCOSII from PNB as behest because of NOCOSII's insufficient capital and inadequate collaterals
after it had examined and studied all the documents relative to the said loan transactions.
Based on the Sworn Statement of PCGG consultant Orlando Salvador, petitioner filed with the Office of the Ombudsman
the criminal complaint against respondents on April 5, 1995. Petitioner alleges that respondents violated the following
provisions of Section 3 (e) and (g) of R.A. No. 3019.
The respondents failed to submit any responsive pleading before the Ombudsman, prompting Graft Investigator Officer
(GIO) I Melinda S. Diaz-Salcedo to resolve the case based on the available evidence. In a Resolution, GIO Diaz-Salcedo
recommended the dismissal of the case on the ground of insufficiency of evidence or lack of probable cause against the
respondents and for prescription of the offense. Ombudsman Desierto approved the recommendation. Petitioner filed a
Motion for Reconsideration but it was denied by GIO Diaz-Salcedo, which was approved by Ombudsman Desierto.
ISSUE: Whether or not the Ombudsman committed grave abuse of discretion in ruling that the offense leveled against
respondents has prescribed.

SOCIAL SECURITY SYSTEM G.R. No. 158131


vs
DEPARTMENT OF JUSTICE
August 8, 2007

RULING
The Supreme Court ruled that SENCOR’s criminal liability was not extinguished. The SC argued that for novation to apply
there must be an original contract to speak of. In this case, the novation does not apply because there was no original
contract that can be replaced by a new contract changing the object or principal condition of the original contract,
substituting the person of the debtor, or subrogating a third person in the rights of the creditor.

The Court observed that although novation is not one of the means recognized by the Revised Penal Code to extinguish
criminal liability, it may prevent the rise of criminal liability or to cast doubt on the true nature of the original basic
transaction, provided the novation takes place before the filing of the Information with the trial court. In the case People v.
Nery, We held:

The novation theory may perhaps apply prior to the filing of the criminal information in court by
the state prosecutors because up to that time the original trust relation may be converted by the parties
into an ordinary creditor-debtor situation, thereby placing the complainant in estoppel to insist on the
original trust. But after the justice authorities have taken cognizance of the crime and instituted action in
court, the offended party may no longer divest the prosecution of its power to exact the criminal liability,
as distinguished from the civil. The crime being an offense against the state, only the latter can renounce
it.
It may be observed in this regard that novation is not one of the means recognized by the Penal
Code whereby criminal liability can be extinguished; hence, the role of novation may only be to either
prevent the rise of criminal liability or to cast doubt on the true nature of the original basic transaction,
whether or not it was such that its breach would not give rise to penal responsibility, as when money
loaned is made to appear as a deposit, or other similar disguise is resorted to.

FACTS
respondent Martels are directors of SENCOR, an information technology firm. Petitioner is a government-owned and
controlled corporation mandated by its charter, RA 1161, to provide financial benefits to private sector employees.
Petitioner filed with the Pasay City Prosecutors Office a complaint against Martels and their five co-accused for SENCORs
non-payment of contributions. To pay this amount, respondent Martels offered to assign to petitioner a parcel of land in
Tagaytay City. Petitioner accepted the offer subject to the condition that respondent Martels will settle their obligation
either by way of dacion en pago or through cash settlement within a reasonable time. Thus, petitioner withdrew its
complaint but reserved its right to revive the same in the event that no settlement is arrived at. In December 2001,
respondent Jose V. Martel wrote petitioner offering, in lieu of the Tagaytay City property, computer-related services.
Petitioner filed with the Pasay City Prosecutors Office another complaint against respondent Martels for SENCORs non-
remittance of contributions.

ISSUE
Whether or not the concept of novation serves to abate the criminal liability of the respondents

Part I, Case No. 70

BENJAMIN ("KOKOY") T. ROMUALDEZ, petitioner, vs.HON. SIMEON V. MARCELO, in his official capacity as the
Ombudsman, and PRESIDENTIAL COMMISSION ON GOOD GOVERNMENT, respondents.
G.R. Nos. 165510-33, July 28, 2006

Intro/Doctrine/Ruling:

Sec. 7 and 11 of RA 3019; Sec. 2 of Act No. 3326;


Article 91 of the RPC cannot be applied suppletorily to Act No. 3326 because such Act is not a special law within the ambit
of Article 10 of the RPC.

1. Yes. Section 11 of RA No. 3019 provides that all offenses punishable therein shall prescribe in 15 years. But prior
to the amendment of Section 11 of R.A. No. 3019 by B.P. Blg. 195 which was approved on March 16, 1982, the
prescriptive period for offenses punishable under the said statute was only ten (10) years. Thus, for offenses allegedly
committed by the petitioner from 1962 up to March 15, 1982, the same shall prescribe in 10 years. On the other hand, for
offenses allegedly committed by the petitioner during the period from March 16, 1982 until 1985, the same shall prescribe
in 15 years.

As to when these two periods begin to run, reference is made to Act No. 3326 which governs the computation of
prescription of offenses defined by and penalized under special laws. Thus, this Court rules that the prescriptive period of
the offenses herein began to run from the discovery thereof or on May 8, 1987, which is the date of the complaint filed by
the former Solicitor General Francisco I. Chavez against the petitioner with the PCGG.

Therefore, when the Office of the Special Prosecutor initiated the preliminary investigation of Criminal Case Nos. 13406-
13429 on March 3, 2004 by requiring the petitioner to submit his counter-affidavit, the alleged offenses subject therein
have already prescribed.

2. No. Under Section 2 of Act No. 3326, the prescriptive period shall be interrupted when proceedings are instituted
against the guilty person. However, there is no such proceeding instituted against the petitioner to warrant the tolling of
the prescriptive periods of the offenses charged against him. In Romualdez v. Sandiganbayan, petitioner averred that
PCGG acted without jurisdiction and/or grave abuse of discretion in conducting a preliminary investigation of cases not
falling within its competence. This Court, in its resolve to deal with the merits of the case to remove the possibility of any
misunderstanding as to the course which it wishes petitioners cases in the Sandiganbayan to take declared invalid the
preliminary investigation conducted by the PCGG over the 24 offenses ascribed to Romualdez.

In Romualdez v. Sandiganbayan, petitioner assailed the validity of the informations filed with the Sandiganbayan in
Criminal Case Nos. 13406-13429 considering that the same were subscribed and filed by the PCGG. In granting
petitioners plea, this Court held, thus:

Here, the informations were filed by an unauthorized party. The defect cannot be cured by conducting
another preliminary investigation. An invalid information is no information at all and cannot be the basis for
criminal proceedings.

Indeed, the nullity of the proceedings initiated by then Solicitor General Chavez in 1987 with the PCGG and by the PCGG
with the Sandiganbayan in 1989 is judicially settled. In contemplation of the law, no proceedings exist that could have
merited the suspension of the prescriptive periods.

3. No. Section 2 of Act No. 3326 provides that the prescription shall begin to run from the day of the commission of
the violation of the law, and if the same be not known at the time, from the discovery thereof and the institution of judicial
proceedings for its investigation and punishment. The running of the prescriptive period shall be interrupted when
proceedings are instituted against the guilty person, and shall begin to run again if the proceedings are dismissed for
reasons not constituting jeopardy. Clearly, Section 2 of Act No. 3326 did not provide that the absence of the accused from
the Philippines prevents the running of the prescriptive period. Thus, the only inference that can be gathered from the
foregoing is that the legislature, in enacting Act No. 3326, did not consider the absence of the accused from the
Philippines as a hindrance to the running of the prescriptive period. Expressio unius est exclusio alterius.

Also, while it is true that Article 10 of the Revised Penal Code makes the Code suppletory to special laws, Act No. 3326
cannot fall within the ambit of special law as contemplated and used in Article 10 of the RPC because it does not define
any acts which are punishable and provides penalties for them. Special law is defined as penal laws that punish acts not
defined and penalized by the Penal Code of the Philippines.

Thus, Art. 91 cannot apply suppletorily to Act No. 3326.

Facts: Petitioner is being charged with violations of Section 7 of RA No. 3019 for failure to file his Statements of Assets
and Liabilities for the period 1967-1985 during his tenure as Ambassador Extraordinary and Plenipotentiary and for the
period 1963-1966 during his tenure as Technical Assistant in the DFA. He claims that the court should dismiss the criminal
cases pending against him on the ground of prescription.

The Ombudsman and the PCGG argue that that the filing of the complaint with the Presidential Commission on Good
Government (PCGG) in 1987 and the filing of the information with the Sandiganbayan in 1989 interrupted the prescriptive
period; that the absence of the petitioner from the Philippines from 1986 until 2000 also interrupted the aforesaid period
based on Article 91 of the Revised Penal Code; that considering that both RA No. 3019 and Act No. 3326 or the Act To
Establish Periods of Prescription For Violations Penalized By Special Acts and Municipal Ordinances and to Provide
When Prescription Shall Begin To Run, are silent as to whether prescription should begin to run when the offender is
absent from the Philippines, the Revised Penal Code, which answers the same in the negative, should be applied.

Issues: 1. Whether the offenses for which petitioner are being charged have already prescribed?
2. WON the filing of the complaint with the PCGG and Sandiganbayan interrupted the running of the prescriptive
period?
3. WON Article 91 of the RPC applies suppletorily to Act No. 3326, such that it suspended the running of the
prescriptive period of 10 or 15 years in RA 3019?

Case #72.
People of the Philippines vs Vergara
GR No 177763; July 3, 2013
Doctrines:
Self-Defense, elements: (1) unlawful aggression on the part of the victim; (2) reasonable necessity of the means
employed to prevent or repel such aggression; and (3) lack of sufficient provocation on the part of the person resorting to
self-defense.
A person who invokes self-defense has the burden of proof. He must prove all the elements of self-defense. However, the
most important of all the elements is unlawful aggression on the part of the victim.
Unlawful aggression must be proved first in order for self-defense to be successfully pleaded, whether complete or
incomplete. Unlawful aggression is an actual physical assault, or at least a threat to inflict real imminent injury, upon a
person. In case of threat, it must be offensive and strong, positively showing the wrongful intent to cause injury. It
"presupposes actual, sudden, unexpected or imminent danger - not merely threatening and intimidating action." It is
present "only when the one attacked faces real and immediate threat to one’s life."
Treachery, how manifested: the offender commits any of the crimes against persons, employing means, methods, or
forms in the execution, which tend directly and specially to insure its execution, without risk to the offender arising from the
defense which the offended party might make.
Article 248. Murder. - Any person who, not falling within the provisions of Article 246, shall kill another, shall be guilty of
murder and shall be punished by reclusion perpetua to death if committed with any of the following attendant
circumstances:

1) With treachery, taking advantage of superior strength, with the aid of armed men, or employing means to weaken the
defense or of means or persons to insure or afford impunity.

Credibility of Witnesses:
When it comes to the matter of credibility of a witness, settled are the guiding rules some of which are that (1) the
Appellate court will not disturb the factual findings of the lower Court, unless there is a showing that it had overlooked,
misunderstood or misapplied some fact or circumstance of weight and substance that would have affected the result of
the case, which showing is absent herein; (2) the findings of the Trial Court pertaining to the credibility of a witness is
entitled to great respect since it had the opportunity to examine his demeanor as he testified on the witness stand, and,
therefore, can discern if such witness is telling the truth or not; and (3) a witness who testifies in a categorical,
straightforward, spontaneous and frank manner and remains consistent on cross-examination is a credible witness.
Facts: Accused Vergara and Inocencio were charged of murder qualified by treachery. Victim was allegedly stabbed by
the accused while the latter were causing some disturbance in the street. Exchange of words lead to the incident initiated
by the accused and the retort of the victim was not likened by the former.
Issues:
1. Whether or not the accused had basis to interpose self-defense? No, the accused had no basis to interpose self-
defense. Elements are not all present. In fact, the unlawful aggression came from the accused and not the victim. The
victim was passing in peace when they approached him and initiated the act of killing causing the life of the victim in peril.
2. Whether or not treachery is present? Yes, treachery is present. Here, accused-appellant Vergara after exchanging
words with the victim, threw his arm around the victim’s shoulder and proceeded to stab him. The victim was totally
unaware of the evil that would befall him. The number and severity of the wounds received by the victim indicated that he
was rendered immobile and without any real opportunity to defend himself other than feebly raising his arm to ward off the
attack.

DAYAP VS SENDIONG

Article 365 of the Revised Penal Code punishes any person who, by reckless imprudence, commits any act which, had
it been intentional, would constitute a grave felony, with the penalty of arresto mayor in its maximum period to prision
correccional in its medium period. When such reckless imprudence the use of a motor vehicle, resulting in the death of a
person attended the same article imposes upon the defendant the penalty of prision correccional in its medium and
maximum periods;

Where a reckless, imprudent, or negligent act results in two or more grave or less grave felonies, a complex
crime is committed. Article 48 of the Revised Penal Code provides that when the single act constitutes two or more
grave or less grave felonies, or when an offense is a necessary means for committing the other, the penalty for the most
serious crime shall be imposed, the same to be applied in its maximum period. Since Article 48 speaks of felonies, it is
applicable to crimes through negligence in view of the definition of felonies in Article 3 as “acts or omissions punishable by
law” committed either by means of deceit (dolo) or fault (culpa). Thus, the penalty imposable upon petitioner, were he to
be found guilty, is prision correccional in its medium period (2 years, 4 months and 1 day to 4 years) and maximum period
(4 years, 2 months and 1 day to 6 years).

FACTS:
Dayap allegedly drove in a reckless manner a 10-wheeler cargo truck hitting an automobile driven by Sendiong who was
with two female passengers. Such incident caused the death of Sendiong, less serious physical injuries on the bodies of
the two female passengers and extensive damage to the automobile. Hence, Dayap was charged with the crime of
Reckless Imprudence resulting to Homicide, Less Serious Physical Injuries, and Damage to Property.

ISSUE:
WON the RTC has jurisdiction to hear a criminal case involving complex crimes such as reckless imprudence resulting in
homicide, less serious physical injuries and damage to property?

RULING:
NO. When this case was filed on 29 December 2004, Section 32(2) of Batas Pambansa Bilang 129 had already been
amended by R.A. No. 7691. R.A. No. 7691 extended the jurisdiction of the first-level courts over criminal cases to include
all offenses punishable with imprisonment not exceeding six (6) years irrespective of the amount of fine, and regardless of
other imposable accessory or other penalties including those for civil liability. It explicitly states "that in offenses involving
damage to property through criminal negligence, they shall have exclusive original jurisdiction thereof." It follows that
criminal cases for reckless imprudence punishable with prision correccional in its medium and maximum
periods should fall within the jurisdiction of the MTC and not the RTC. Clearly, therefore, jurisdiction to hear and try
the same pertained to the MTC and the RTC did not have original jurisdiction over the criminal case. Consequently, the
MTC of Sibulan, Negros Oriental had properly taken cognizance of the case and the proceedings before it were valid and
legal.

People vs Mamantak
GR no. 174659
July 28, 2008

Article 267 of the RPC

Facts:

Christopher, a two-year old minor, had disappeared from mcdonalds in Binondo when his mom and his sister were
ordering from the counter. On February 25, 2001, Teresa received a call from a woman and claimed to have custody of
Christopher and asked forP30,000 in exchange for the boy. They then agreed to conduct the pay off in the morning of April
7, 2001 at Pitang’s Carinderia in Kapatagan, Lanao del Norte.

Issue:
Whether the two accused are guilty of violating the crime of Kidnapping for Ransom under Article 267 ofthe RPC, as
amended by RA No. 7659?

Ruling: Yes
Kidnapping; Elements.—The crime has the following elements: (1) the offender is a private individual; not either of the
parents of the victim or a public officer who has a duty under the law to detain a person; (2) he kidnaps or detains another,
or in any manner deprives the latter of his liberty; (3) the act of detention or kidnapping must be illegal and (4) in the
commission of the offense, any of the following circumstances is present: (a) the kidnapping or detention lasts for more
than three days; (b) it is committed by simulating public authority; (c) any serious physical injuries are inflicted upon the
person kidnapped or detained or threats to kill him are made or (d) the person kidnapped or detained is a minor, female or
a public official.

The essence of the crime of kidnapping is the actual deprivation of the victim’s liberty coupled with the intent of the
accused to effect it; If the victim is a minor, or the victim is kidnapped and illegally detained for the purpose of extorting
ransom, the duration of his detention becomes inconsequential.

SAFEGUARD SECURITY vs. TANGCO


GR No. 165732, December 14, 2006
Austria Martinez, J.:

DOCTRINE: An act or omission causing damage to another may give rise to two separate civil liabilities on the part of the
offender, i.e., (1) civil liability ex delicto, under Article 100 of the Revised Penal Code; and (2) independent civil liabilities,
such as those (a) not arising from an act or omission complained of as a felony, e.g., culpa contractual or obligations
arising from law under Article 31 of the Civil Code, intentional torts under Articles 32 and 34, and culpa aquiliana under
Article 2176 of the Civil Code; or (b) where the injured party is granted a right to file an action independent and distinct
from the criminal action under Article 33 of the Civil Code. Either of these liabilities may be enforced against the offender
subject to the caveat under Article 2177 of the Civil Code that the offended party cannot recover damages twice for the
same act or omission or under both causes.
Provisions mentioned: Art. 2176, 2177 of the Civil code, Article 103 of the RPC
FACTS:
On November 3, 1997, at about 2:50 p.m., Evangeline Tangco (Evangeline) went to Ecology Bank, Katipunan Branch,
Quezon City, to renew her time deposit per advise of the bank's cashier as she would sign a specimen card. Evangeline, a
duly licensed firearm holder with corresponding permit to carry the same outside her residence, approached security
guard Pajarillo, who was stationed outside the bank, and pulled out her firearm from her bag to deposit the same for
safekeeping. Suddenly, Pajarillo shot Evangeline with his service shotgun hitting her in the abdomen instantly causing her
death.
Lauro Tangco, Evangeline's husband, together with his six minor children (respondents) filed with the Regional Trial Court
(RTC) of Quezon City, a criminal case of Homicide against Pajarillo, docketed as Criminal Case No. 0-97-73806 and
assigned to Branch 78. Respondents reserved their right to file a separate civil action in the said criminal case. The RTC
of Quezon City subsequently convicted Pajarillo of Homicide in its Decision dated January 19, 2000. On appeal to the CA,
the RTC decision was affirmed with modification as to the penalty in a Decision.
Meanwhile, on January 14, 1998, respondents filed with RTC, Branch 273, Marikina City, a complaint5 for damages
against Pajarillo for negligently shooting Evangeline and against Safeguard for failing to observe the diligence of a good
father of a family to prevent the damage committed by its security guard. Respondents prayed for actual, moral and
exemplary damages and attorney's fees.
In their Answer,6 petitioners denied the material allegations in the complaint and alleged that Safeguard exercised the
diligence of a good father of a family in the selection and supervision of Pajarillo; that Evangeline's death was not due to
Pajarillo's negligence as the latter acted only in self-defense.
The RTC found respondents to be entitled to damages. It rejected Pajarillo's claim that he merely acted in self-defense. It
gave no credence to Pajarillo's bare claim that Evangeline was seen roaming around the area prior to the shooting
incident since Pajarillo had not made such report to the head office and the police authorities. The RTC further ruled that
being the guard on duty, the situation demanded that he should have exercised proper prudence and necessary care by
asking Evangeline for him to ascertain the matter instead of shooting her instantly; that Pajarillo had already been
convicted of Homicide in Criminal Case No. 0-97-73806; and that he also failed to proffer proof negating liability in the
instant case.

The RTC also found Safeguard as employer of Pajarillo to be jointly and severally liable with Pajarillo. It ruled that while it
may be conceded that Safeguard had perhaps exercised care in the selection of its employees, particularly of Pajarillo,
there was no sufficient evidence to show that Safeguard exercised the diligence of a good father of a family in the
supervision of its employee; that Safeguard's evidence simply showed that it required its guards to attend trainings and
seminars which is not the supervision contemplated under the law; that supervision includes not only the issuance of
regulations and instructions designed for the protection of persons and property, for the guidance of their servants and
employees, but also the duty to see to it that such regulations and instructions are faithfully complied with.
Petitioners appealed the RTC decision to the CA. On July 16, 2004, the CA issued its assailed Decision, the dispositive
portion of which reads:
IN VIEW OF ALL THE FOREGOING, the appealed decision is hereby AFFIRMED, with the modification that Safeguard
Security Agency, Inc.'s civil liability in this case is only subsidiary under Art. 103 of the Revised Penal Code.
In finding that Safeguard is only subsidiarily liable, the CA held that the applicable provisions are not Article 2180 in
relation to Article 2176 of the Civil Code, on quasi-delicts, but the provisions on civil liability arising from felonies under the
Revised Penal Code; that since Pajarillo had been found guilty of Homicide in a final and executory judgment and is said
to be serving sentence in Muntinlupa, he must be adjudged civilly liable under the provisions of Article 100 of the Revised
Penal Code since the civil liability recoverable in the criminal action is one solely dependent upon conviction, because
said liability arises from the offense charged and no other; that this is also the civil liability that is deemed extinguished
with the extinction of the penal liability with a pronouncement that the fact from which the civil action might proceed does
not exist; that unlike in civil liability arising from quasi-delict, the defense of diligence of a good father of a family in the
employment and supervision of employees is inapplicable and irrelevant in civil liabilities based on crimes or ex-delicto;
that Article 103 of the Revised Penal Code provides that the liability of an employer for the civil liability of their employees
is only subsidiary, not joint or solidary.
ISSUE/S:
Whether or not the CA erred in ruling that the liability of Safeguard Security is subsidiary
HELD:
YES. Safeguard insists that the claim for damages by respondents is based on culpa aquiliana under Article 217611 of the
Civil Code, in which case, its liability is jointly and severally with Pajarillo. However, since it has established that it had
exercised due diligence in the selection and supervision of Pajarillo, it should be exonerated from civil liability.
The law at the time the complaint for damages was filed is Rule 111 of the 1985 Rules on Criminal Procedure, as
amended, to wit:

SECTION 1. Institution of criminal and civil actions. - When a criminal action is instituted, the civil action for the
recovery of civil liability is impliedly instituted with the criminal action, unless the offended party waives the civil
action, reserves his right to institute it separately, or institutes the civil action prior to the criminal action.
Such civil action includes recovery of indemnity under the Revised Penal Code, and damages under Articles 32,
33, 34, and 2176 of the Civil Code of the Philippines arising from the same act or omission of the accused.
Respondents reserved the right to file a separate civil action and in fact filed the same on January 14, 1998.
The CA found that the source of damages in the instant case must be the crime of homicide, for which he had already
been found guilty of and serving sentence thereof, thus must be governed by the Revised Penal Code
An act or omission causing damage to another may give rise to two separate civil liabilities on the part of the offender, i.e.,
(1) civil liability ex delicto, under Article 100 of the Revised Penal Code; and (2) independent civil liabilities, such as those
(a) not arising from an act or omission complained of as a felony, e.g., culpa contractual or obligations arising from law
under Article 31 of the Civil Code, intentional torts under Articles 32 and 34, and culpa aquiliana under Article 2176 of the
Civil Code; or (b) where the injured party is granted a right to file an action independent and distinct from the criminal
action under Article 33 of the Civil Code. Either of these liabilities may be enforced against the offender subject to the
caveat under Article 2177 of the Civil Code that the offended party cannot recover damages twice for the same act or
omission or under both causes.
A reading of respondents' complaint shows that the latter are invoking their right to recover damages against Safeguard
for their vicarious responsibility for the injury caused by Pajarillo's act of shooting and killing Evangeline under Article
2176, Civil Code. The scope of Article 2176 is not limited to acts or omissions resulting from negligence. Well-entrenched
is the doctrine that Article 2176 covers not only acts committed with negligence, but also acts which are voluntary and
intentional.
In Elcano vs Hill, the court ruled that: Article 2176, where it refers to "fault or negligence," covers not only acts "not
punishable by law" but also acts criminal in character, whether intentional and voluntary or negligent. Consequently, a
separate civil action lies against the offender in a criminal act, whether or not he is criminally prosecuted and found guilty
or acquitted, provided that the offended party is not allowed, if he is actually charged also criminally, to recover damages
on both scores, and would be entitled in such eventuality only to the bigger award of the two, assuming the awards made
in the two cases vary. In other words, the extinction of civil liability referred to in Par. (e) of Section 3, Rule 111, refers
exclusively to civil liability founded on Article 100 of the Revised Penal Code, whereas the civil liability for the same act
considered as quasi-delict only and not as a crime is not extinguished even by a declaration in the criminal case that the
criminal act charged has not happened or has not been committed by the accused. Briefly stated, We here hold, in
reiteration of Garcia, that culpa aquiliana includes voluntary and negligent acts which may be punishable by law.
The civil action filed by respondents was not derived from the criminal liability of Pajarillo in the criminal case but one
based on culpa aquiliana or quasi-delict which is separate and distinct from the civil liability arising from crime. The source
of the obligation sought to be enforced in the civil case is a quasi-delict not an act or omission punishable by law.
Since the civil liability was founded on a quasi-delict and not from the crime of homicide. The liability of Safeguard Security
must be Solidary and not Subsidiary.

G.R. No. 178115 July 28, 2014


PEOPLE OF THE PHILIPPINES, Plaintiff-Appellee,
vs.
JOJO SUMILHIG, RICARDO SUMILHIG alias CARDING SUMILHIG, PASOT SALOLI, ERIC ENOC, WARLITO
MONTEBON,* and CIO LIMAMA, Accused,
JOJO SUMILHIG, RICARDO SUMILHIG alias CARDING SUMILHIG, and PASOT SALOLI, Accused-Appellants.

FACTS: Appellants, together with the accused Eric Enoc, Warlito Montibon and Cio Limama, were charged with double
murder and double frustrated murder in an Amended Information, 3 the accusatory portion of which reads:
That on or about October 31, 1998, inthe Municipality of Kiblawan, Province of Davao del Sur, Philippines, and within the
jurisdiction of this Honorable Court, the above-named accused, conspiring, confederating and helping one another, armed
with assortedfirearms, with intent to kill with treachery and evident premeditation, did, then and there willfully, unlawfully
and feloniously, simultaneously strafe the house of Eugenio Santander resulting to death of [Cresjoy] Santander and
RollySantander and seriously wounding Marissa Santander and Micel Santander, which would have caused their death
had there been no timely and able medical assistance rendered to them, to the damage and prejudice of the offended
parties.

ISSUE:
WON: the crime committed is a complex crime

Rulings:
Ruling of the Regional Trial Court
The RTC convicted the appellants ofthe complex crime of double murder and double frustrated murder and sentenced
them to suffer the penalty of death. It further ordered them to indemnify, jointlyand severally, the heirs of Cresjoy and Rolly
the sum of P100,000.00 as civil indemnity, and the surviving victims, Marissa and Micel, the sums of P50,000.00
and P30,000.00 as moral and exemplary damages, respectively. 8
Ruling of the Court of Appeals
On appeal, the CA did not find any reason to disturb the findings of the RTC. However, it found merit in appellants’
argument that the crime committed could not have been a complex crime since the death and injuries complained of did
not result from a single act but from several and distinct acts of shooting. And as treachery was alleged in the Amended
Information and sufficiently proven during trial, appellants should be convicted instead of two counts of murder and two
counts of frustrated murder. Thus, in rendering its Decision 9 dated July 28, 2006, the CA disposed ofthe case as follows:
WHEREFORE, the judgment of conviction of appellants Jojo Sumilhig, Alias Carding Sumilhig and Pasot Saloli is affirmed
butmodified as follows –
Appellants Juan "Jojo" Sumilhig, Alias Carding Sumilhig and Pasot Saloli, are found guilty beyondreasonable doubt of:
a. Murder for killing Crisjoy Santander, and x x x are sentenced to suffer the penalty of reclusion perpetua;
b. Murder for the killing of Rolly Santander, and x x x are sentenced to suffer the penalty of reclusion perpetua;
c. Frustrated Murder for the shooting of Marissa Santander and x x x are sentenced to suffer an imprisonment
ofsix (6) years, four (4) months and [ten] (10) days of prision mayoras minimum to fourteen (14) years, eight (8)
months and twenty (20) days of reclusion temporalas maximum; d. Frustrated Murder for the shooting of Micel
Santander and x x x are sentenced to suffer an imprisonment ofsix (6) years, four (4) months and ten (10) days of
prision mayoras minimum to fourteen (14) years, eight (8) months and twenty (20) days of reclusion temporalas
maximum. All accused are ordered to indemnify jointly and severally the heirs of Crisjoy Santander and Rolly
Santander the sum of P100,000.00 and the surviving victims Marissa Santander and Micel Santander P50,000.00
as moral damages and P30,000.00 as exemplary damages, without subsidiary imprisonment in case of
insolvency.
Ruling of the Supreme Court
The appeal has no merit.
Appellants’ conviction was based on
their positive identification by the
prosecution witnesses.
True, the RTC noted in its Decision the existence of motive on the part of Jojo for committing the crime as well as Pasot’s
incredulous claim of ignorance on almost about everything. It is well to note, however, that the said court neither based
the appellants’ conviction on the existence of such motive nor on Pasot’s weak defense of ignorance alone, but upon the
prosecution witnesses’ identification of appellants as the assailants, viz:
Assessing the evidence presented by both [P]rosecution and defense, we see a less than glaring hint of vendetta. As part
of his defense, the accused Jojo Sumilhig narrated that his family was massacred by Jerry Santander, brother of Remigio
Santander [in] February 1998. Short of admitting the crime, Sumilhig stated that because of this, he harbored ill feelings
not only against Jerry and Carlos Santander but also against their family. Thus a clear motive for killing the Santander
family has been established giving credence to prosecution witnesses’ allegation that after the strafing Jojo Sumilhig
shouted"Nakabalos na ko!" The likelihood of his intention to wipe out the said family became even more apparent.
Despite his positive assertion that it was the Santanders that killed his family, he did not file any case against them. It was
only after he was arrested that he filed a complaint against Jerry and Carlos Santander.
His alibi likewise failed to meet the stringent requisites of the Rules. Even as Dr. Quirapas appeared determined to rule
out the possibility that he could walk without crutches five months after his discharge, the same was based on general
medical prognosis. Such prognostication admits certain exception[s], as could be gleaned from the testimony of the doctor
himself that the healing period may vary depending on the age and physical condition of the patient. Notably Jojo Sumilhig
was then 23 years old.
What was certain was the positive identification made by Jerry Masaglang and Remegio Santander of all of the accused.
The "overkill" by which the accused Pasot Saloli and Carding Sumilhig claimed total ignorance of almost anything only
served to arouse incredulity. Both accused claimed they could not read, write, tell time, day, month or year. Neither could
[they] allegedly speak [or] understand Visayan, which is of common knowledge to be widely spoken in almost every part of
Mindanao. Saloli claimed he did not know what day [it] was when he was testifying, or the day before and after that. Both
claimed they did not know the complainants or of the massacre that took place.
xxxx
More importantly, these claims [of] utter ignorance are belied by the evasiveness by which all three accused answered in
obvious effort to avoid criminal responsibility. Behind the façade of ignorance and lack of education lurks a calculating
mind. We find [it] difficult to ascribe innocence to the accused when traces of ingenuity and craftiness characterize their
testimonies.
All these observations however become insignificant in the face of the positive and spontaneous identification of the
assailants/accused by credible witnesses Jerry Masaglang and Remigio Santander. 15
There is no reason to doubt Jerry and Mario’s identification of the appellants considering that (1) Jerry was just six meters
away from them;16 (2) the moon was bright and Jerry was familiar with all the accused as most of them are his
relatives;17 and, (3) Mario knows Jojoever since he was small. 18 Besides, "[t]ime-tested is the rulethat between the positive
assertions of prosecution witnesses and the negative averments of the accused, the former undisputedly [deserve] more
credence and [are] entitled to greater evidentiary weight." 19
Anent the respective alibis interposed by appellants, suffice it to say that "[a]libi cannot prevail over the positive
identification of a credible witness."20
There was conspiracy among the accused.
"[C]onspiracy exists when two or more persons come to an agreement concerning the commission of a felony and decide
to commit it."21 It is not necessary to adduce evidence of a previous agreement to commit a crime. 22"Conspiracy may be
shown through circumstantial evidence, deduced from the mode and manner in which the offense was perpetrated, or
inferred from the acts of the accused themselves when such leadto a joint purpose and design, concerted action, and
community of interest."23
Here, there is no proof of a previous agreement among the accused but there is a series of events that clearly established
conspiracy among them. First, they were all armed with firearms. Second, they surreptitiously approached the crime
scene. Third, when they were within close range of the intended victims, they simultaneously discharged their firearms.
Fourth, they ceased firing at the same time and fled together. Undoubtedly, their acts before, during and immediately after
strafing the house of Eugenio evince their unanimity in design, intent and execution. 24 Treachery attended the commission
of the crime.
"There is treachery whenthe offender commits any of the crimes against the person, employing means, methods orforms
in the execution thereof which tend directly and specially to insure the execution, without risk to himself arising from [any]
defense which the offended party might make." 25
Treachery is evident in this case as the suddenness and unexpectedness of the assault deprived the victims of an
opportunity to resist it or offer any defense of their persons. This is considering that the victims were unaware that they
would be attacked by appellants with a hailof bullets from their firearms fired at close range. Indeed, "[t]he suddenness of
the attack, without the slightest forewarning thereof, placed the [victims] x x x in such a position that they could not have
defended themselvesfrom the aggression x x x."26
The crime committed is two counts of murder and two counts of frustrated murder.

PEOPLE OF THE PHILIPPINES, Plaintiff-Appellee, v. ADRIAN GUTING Y TOMAS, Accused-Appellant.


G.R. No. 205412, September 09, 2015 LEONARDO-DE CASTRO, J.

DOCTRINE:
Article 246 of the Revised Penal Code defines Parricide as follows:

Art. 246. Parricide. - Any person who shall kill his father, mother, or child, whether legitimate or illegitimate, or any of his
ascendants, or descendants, or his spouse, shall be guilty of parricide and shall be punished by the penalty of reclusion
perpetua to death.

Parricide is committed when: (1) a person is killed; (2) the deceased is killed by the accused; and (3) the deceased is the
father, mother, or child, whether legitimate or illegitimate, or a legitimate other ascendant or other descendant, or the
legitimate spouse of the accused. The key element in Parricide - other than the fact of killing - is the relationship of the
offender to the victim.23 All the elements are present in this case. Jose, the victim, was killed by accused-appellant, his
own son.

The crime of Parricide is punishable by the indivisible penalties of reclusion perpetua to death. With one mitigating
circumstance, namely, voluntary surrender, and no aggravating circumstance, the imposition of the lesser penalty
of reclusion perpetua on accused-appellant was proper.

FACTS:
The said accused, did then and there willfully, unlawfully and feloniously, and with evident premeditation, that is, having
conceived and deliberated to kill his own father Jose Guting y Ibarra, 67 years old, married, while inside their residential
house, and armed with a bladed weapon, suddenly and unexpectedly stabbed several times the victim, employing means,
manner and form in the execution thereof which tender directly and specially to insure its commission without danger to
the person of said accused, the result of which attack was that said victim received multiple stab wounds on his body
which directly caused his instantaneous death.

The RTC promulgated its Decision on June 24, 2010 finding accused-appellant guilty of Parricide based on his verbal
admission that he killed his father, Jose. Even assuming that accused-appellant's admission was inadmissible in
evidence, the RTC adjudged that the prosecution was still able to establish sufficient circumstantial evidence which, taken
collectively, pointed to accused-appellant as the perpetrator of the brutal killing of his father.

ISSUE:
Whether the accused-appellant guilty beyond reasonable doubt of the crime charged despite the prosecution's failure to
overthrow the constitutional presumption of innocence in his favor.

Vous aimerez peut-être aussi